Jump to content

Wikipedia:Reference desk/Humanities

From Wikipedia, the free encyclopedia

This is an old revision of this page, as edited by 142.68.216.139 (talk) at 13:25, 15 March 2009 (→‎Canadian equivalent of "Datuk": new section). The present address (URL) is a permanent link to this revision, which may differ significantly from the current revision.

Welcome to the humanities section
of the Wikipedia reference desk.
Select a section:
Want a faster answer?

Main page: Help searching Wikipedia

   

How can I get my question answered?

  • Select the section of the desk that best fits the general topic of your question (see the navigation column to the right).
  • Post your question to only one section, providing a short header that gives the topic of your question.
  • Type '~~~~' (that is, four tilde characters) at the end – this signs and dates your contribution so we know who wrote what and when.
  • Don't post personal contact information – it will be removed. Any answers will be provided here.
  • Please be as specific as possible, and include all relevant context – the usefulness of answers may depend on the context.
  • Note:
    • We don't answer (and may remove) questions that require medical diagnosis or legal advice.
    • We don't answer requests for opinions, predictions or debate.
    • We don't do your homework for you, though we'll help you past the stuck point.
    • We don't conduct original research or provide a free source of ideas, but we'll help you find information you need.



How do I answer a question?

Main page: Wikipedia:Reference desk/Guidelines

  • The best answers address the question directly, and back up facts with wikilinks and links to sources. Do not edit others' comments and do not give any medical or legal advice.
See also:


March 9

Why Is Only Saturday Named After a Roman God?

As the article on Saturday notes, Saturday is the only day of the week in which the English name comes from Roman mythology; the English names of all of the other days of the week come from Anglo-Saxon polytheism. Why did Saturday alone receive this distinction? Saturn doesn't seem like an obvious choice of namesakes at all. I assume we got "Saturday" from the Anglo-Saxon (Germanic) settlers of Britain (it's akin to Old Frisian sāterdei, according to Merriam-Webster), so why would there even have been a Latin influence on something as basic as the days of the week? And if there were a Latin influence, why did it not extend to the names of the other days? John M Baker (talk) 00:02, 9 March 2009 (UTC)[reply]

According to our article Saturday was named after the planet and not the god directly. Of course, you are right that it is the only one sharing a name with the Roman gods. Sunday and Monday are of course named after the Sun and Moon, while Tuesday (Tyr), Wednesday (Odin), Thursday (Thor), and Friday (Friga) are all named after Norse Gods. Not sure there's a reason for this beyond "just cuz"... --Jayron32.talk.contribs 00:51, 9 March 2009 (UTC)[reply]
Pushing it back a level doesn't make much difference. Mercury, Venus, Mars, and Jupiter are all planets named after Roman gods, but none of them have their own days. Also, I note that the article's assertion that the day is named for the planet is uncited, so I'm not sure how much credence to give it. John M Baker (talk) 01:00, 9 March 2009 (UTC)[reply]
Probably because of England's Norse heritage. Other Romanized (not sure if I spelled that correctly) countries in southern Europe might use names of Roman gods. --Queen Elizabeth II's Little Spy (talk) 00:56, 9 March 2009 (UTC)[reply]
Not "might", "do". For example, the French for Tuesday is "mardi", named after the Roman God/planet Mars. --Tango (talk) 01:10, 9 March 2009 (UTC)[reply]
John M -- The week was spread to northern Europe through the Roman Empire, so of course it wouldn't be surprising for Latin to exert an influence. None of the West Germanic languages use the name of a native god for Saturday (high German uses a very altered form of the word "Sabbath", while the other languages use a form of "Saturn day"), so maybe no god was considered equivalent to Roman Saturnus, or was assigned to the planet. The North Germanic languages use forms related to "Laugardagur" for Saturday... AnonMoos (talk) 02:15, 9 March 2009 (UTC)[reply]

I see that there is excellent information in the article on week-day names, to which I've added a link from Saturday. If I'm following the discussion there properly, actually all of the weekday names are translations from Latin, but with cognate Norse gods substituted for the original Roman gods (a somewhat strained cognate in the case of Mercury/Woden), and apparently there was not thought to be any cognate for Saturn. The assertion in the Saturday article, that the English names of all of the other days of the week come from Anglo-Saxon polytheism, is apparently a bit misleading, particularly in the cases of Sunday and Monday. John M Baker (talk) 04:13, 9 March 2009 (UTC)[reply]

I find it curious that when the Catholic Church held sway over all of Europe, they didn't declare that it was blasphemous to use names for days (or months) which were named after non-Christian gods. I'd expect they would have provided us with names like Johnday, Josephday, Maryday, Mosesday, Solomanday, Adamday, and, of course, Jesusday. StuRat (talk) 01:00, 10 March 2009 (UTC)[reply]
StuRat -- look at Franz Werfel's strange (but kind of interesting) semi-classic science fiction novel "Star of the Unborn" for a hypothetical Christianized version of the names of the planets (I can't remember them all, but Venus was Mary Magdalene, and I seem to remember that Jupiter was John the Baptist). Some of the names of the Roman calendar months originally had connections with Roman pagan religion, but they also survived the coming of Christianity... AnonMoos (talk) 06:51, 10 March 2009 (UTC)[reply]
Interesting. StuRat (talk) 22:12, 10 March 2009 (UTC)[reply]
The Quakers used to avoid using "pagan" names for days of the week and instead just called them First Day, Second Day, etc. -- Mwalcoff (talk) 02:27, 10 March 2009 (UTC)[reply]
Once these names take hold, they stick. When the Bolshevik atheists came to power in Russia, they didn't change the Russian name for Saturday just because it's cognate with "Sabbath" (Суббота - Subbóta). Nor that of Sunday (Воскресенье - Voskresénye - which literally means "Resurrection"). -- JackofOz (talk) 04:05, 10 March 2009 (UTC)[reply]
It seems like, during the Spanish Inquisition, they endeavored to wipe out entire religions and cultures in Spain, so forcing people to use new names for the days of the week, months, and planets would be relatively easy, if they could burn anyone alive who refused. I'd have to conclude that changing those names just wasn't a priority with the leaders of the Inquisition. Perhaps they considered other religions and cultures to be far more of a threat to their power base. StuRat (talk) 18:54, 10 March 2009 (UTC)[reply]
It's a mystery. Saturn, the deity, is glossed in Old Norse sources with the North Germanic god Njörðr, so one would think Njörðr would be the nearest equivalent. Njörðr's importance is well attested among the North Germanic peoples, and his female "twin" is attested in Tacitus' 1st century work Germania; Nerthus. Therefore Nerthus would have been known to the Angles—who later became the West Germanic Anglo-Saxons that formed England—and the North Germanic Danes. So why didn't they gloss Saturday as "Njörðr's day"? While the connection may be obvious with the late gloss in mind, I've never seen the Njörðr-Saturday thing mentioned by anyone anywhere, so this is just my personal commentary. Jacob Grimm wrote a bit about how the name may point to a similarly named, unattested figure potentially from Germanic mythology in volume I of his Teutonic Mythology, and this likely deserves some more attention. :bloodofox: (talk) 02:56, 10 March 2009 (UTC)[reply]

Why aren't people buying stocks in mass?

According to the indexes, stock prices have dropped more or less in half. I would think this is a great time to buy stocks, considering that the recession is bound to end some time. Correct me if I'm wrong: Do people wait to buy stocks because they want to wait for it to be at the cheapest price? In other words, are they pouncing only when they think the market is at the bottom?128.163.224.198 (talk) 00:19, 9 March 2009 (UTC)[reply]

People who invest are investing. People who do not invest are, in general, not investing. Asking why people don't invest really has nothing to do with the stock market or the economy. It is mainly just the person. If you give one person a thousand dollars, he might invest it. Another person might off credit cards. Another might race off to Vegas and lose it all on the first night. -- kainaw 01:12, 9 March 2009 (UTC)[reply]
People are not "pouncing", as you put it, despite the low index, because of several reasons. One, no-one can foresee how this current crisis will turn out, or when the market will recover. It may still fall further in some markets. It's a fairly risky time to be investing. It is precisely your confidence ("recession is bound to end some time") that many people lack. Plus, "some time" could be quite some time.
Two, the reason the stocks fell in the first place, if you remember, is because people had to sell, for various reasons including due to their exposure to subprime mortgages. Many of these issues have not been resolved: it is still not easy to find debt financing, for example. As a result, those people who sold out are often still not in a position to buy-in.
Given this, there *are* safer forms of investment which people may prefer. At the moment, a term deposit in a bank is not a bad idea. Others are investing in gold and other commodities. --PalaceGuard008 (Talk) 01:51, 9 March 2009 (UTC)[reply]
No matter what anyone says, investing is a form of gambling. Thus it should only be done with discretionary funds - money one doesn't need for other purposes. With the current financial crisis, many people are worried that their jobs (thus incomes) may disappear, so they hold on to their cash. For those who do have extra funds available, you are correct that this would be a good time to invest, but, as PalaceGuard008 points out, some have lost their discretionary funds, while others are in a "wait-and-see" mode. On the other hand, as per Kainaw, those with extra money at hand and who wish to invest see this period as an opportunity. Then, again, why do you expect everyone to act rationally in the first place?
Let me ask a counter-question. When, in the last six months, have you heard a call for George W Bush's proposal to take half of the Social Security Trust Fund and invest it in the Stock Market because that will give beneficiaries larger retirement accounts?
The Stock Market will come back. The question is "When?" After the 1929 crash it took 25 years for the Dow-Jones to return to pre-crash levels. New Deal regulations and the federal government's quick response to this crisis should shorten the recovery period considerably, but it is not surprising that many people are taking a cautious approach - which is the proper attitude under all circumstances. B00P (talk) 04:22, 9 March 2009 (UTC)[reply]
Theoretically speaking the stock market is not a zero sum game so it's not 'gambling'. The reason people are not buying stocks is much like people have noted. Confidence at the moment is very low - people invest in different 'cycles' - some day-trade, some want a return in 12-24 months, some longer, some just want to buy it and leave it for 20 years. Each person has a different level of confidence in the current price. I've no doubt there are plenty of people picking up stock at these prices and then expecting in 5 years when they return to them that they'll be worth X-times-more. That person is happy tieing up their capital in something that is highly volatile currently and so maybe can only be sold at profit in 3 or 4 years. The person who has 10k they don't need now but might need in 12 months is less likely to invest - but in a positive stable market they might consider it worthwhile to invest for that period. It's all about individual choices all adding up to make up the market sentiment. Beyond that institutional investors will have a huge impact too and they have similar worries but on differing scales. 194.221.133.226 (talk) 14:22, 9 March 2009 (UTC)[reply]
The Wall Street Crash has a salutary lesson for the "I'll get in at the bottom" investor. A lot of those who really lost a lot of money were not those who were invested in the market at the beginning, but those who invested at a "shelf", which they thought was the bottom of a u-shaped depression but was in fact just a temporary station on an ongoing decline. Such investors even caused little rallies (and whatever happens to the modern market, you'll see the same things). The trouble is that you can't tell the difference between the market bottoming out and a temporary shelf, and you can't tell the difference between a genuine beginning to recovery and a (destined to be short lived) false rally caused by others' mistaken optimism. Serious investors study the "value" of the company, which they compute using a bunch of metrics including its P-E ratio; but with the profits of so many companies negative, that's hard to do. Mimetic Polyalloy (talk) 14:26, 9 March 2009 (UTC)[reply]

All investment is a judgement on the balance between risk and reward. If people are not, in your opinion, investing, it is very likely to be because they judge the risks to out-weigh the rewards at these prices. DOR (HK) (talk) 10:04, 9 March 2009 (UTC)[reply]

To answer your question very simply, you could have said exactly the same thing three months ago, and it would have sounded just as convincing. However since then stocks have plummeted even further. DJ Clayworth (talk) 13:48, 9 March 2009 (UTC)[reply]

I agree with the comments made above, but I have this to add: Stocks represent ownership of a fraction of a firm. Therefore, their price reflects investors' perceptions of the value of the firm. That value is relative to its earnings and its expected future earnings and to the dividends that investors might gain from those earnings. As firms' profits have shrunk or turned into losses, the perceived value of those firms has dropped accordingly. Investors might still bid up the price of a firm that they believe is temporarily loss-making if they believe that it is likely to regain profitability. However, a huge question mark hangs over the future of many firms and the economy as a whole. As the economic data steadily worsen, investors fear a severe recession in which many firms might go bankrupt and see their value evaporate. This fear leads many people to shun a market that seems much more risky than it did just two years ago and to try to find a safer haven for their savings. Marco polo (talk) 01:05, 10 March 2009 (UTC)[reply]
Those investors or institutions who are buying in mass will be keeping quiet about it, as if they make it know they are buying prices will rise and they will have to pay more. 89.243.95.28 (talk) 13:43, 15 March 2009 (UTC)[reply]

accented poetry

When a poet writes poetry that counts accented syllables (iambic pentameter, etc.) does the poet consider the secondary stress as part of the rhythm? For example, would the word "universe" be considered a dactyl or something else?

Normally each accented syllable is counted. "Universe" theoretically could be a cretic foot, but typically it would be combined with an unstressed preceding syllable to make two iambs ("the UniVERSE") or with an unstressed following syllable to make two trochees ("UniVERses"). John M Baker (talk) 04:21, 9 March 2009 (UTC)[reply]

What is the female form of the Kayser-i-Rûm?

What is it in Ottoman Turkish? Kayser-i-Rûm means the Emperor/Caesar of Rome. It was claimed by Mehmed II who conquered Constantinople. --Queen Elizabeth II's Little Spy (talk) 01:20, 9 March 2009 (UTC)[reply]

I doubt that Ottomans had any real concept of "empress", as this term has been used in European languages. The Ottoman sultans fully indulged their rights under Islamic law to take four wives and/or an indefinite number of slave concubines, but none of these women were really considered partners in helping the sovereign to rule or had a formalized publicly-visible social role (as was often the case with European royal wives). In fact, the only woman who regularly appeared in public at all was the mother of the reigning sultan, who occasionally decorously appeared on high balconies overlooking certain ceremonies. AnonMoos (talk) 01:53, 9 March 2009 (UTC)[reply]
I notice that also. But I was wondering if there is female form of the title. Were the Mother Sultans a mere copy of the Roman and Byzantine title Empress Mother? --Queen Elizabeth II's Little Spy (talk) 02:06, 9 March 2009 (UTC)[reply]
I think you're seeking equivalences where the predominant characteristics of societies meant that such equivalences didn't really exist. The role of the queen-consort in European societies was consistent with the fact that women at a lower social level had a publicly-visible "hostess" role (meeting men whom her husband invited to their home), and supervised most of the economically productive activities in the household other than heavy agricultural field work, etc. The lack of any real public "queen" or "empress" role in the Ottoman sultanate was consistent with the fact that upper-class women in many Islamic societies were often kept strictly secluded from public view, were not present at social events involving men outside their family, etc. It was not considered a good thing for women to rule in either type of society, but at least a few prominent European women could influence events in a somewhat open above-board and quasi-legitimate manner -- while women exercising political influence in the Ottoman system was always considered illegitimate pernicious boudoir-harem-eunuch intrigues and corruption by definition. AnonMoos (talk) 02:51, 9 March 2009 (UTC)[reply]
For the puroses of this discussion, it might be useful to know the existence of Sultana (title). The article Islamic honorifics needs improvement. BrainyBabe (talk) 08:08, 9 March 2009 (UTC)[reply]

Participants in a survey more/less likely to trust it?

Are people who participated in a survey more or less likely to trust the results than people who could have been selected to participate but weren't? NeonMerlin 01:47, 9 March 2009 (UTC)[reply]

I don't believe that it makes any difference. Let us suppose the survey is to determine the relative popularity of odd and even numbers. If you participated and chose "odd," but "even" was more popular, would you be more likely to trust that result than if you hadn't participated? In fact, you might be more dubious, as you know how you voted. On the other hand, if you had voted "even," you would be more likely to trust the results, and for exactly the same reason. However, this is only my opinion; in order to find out, I suggest that you take a survey. <!> —B00P (talk) 04:38, 9 March 2009 (UTC)[reply]
I think the answer depends on several factors. Call the dependent quantity the participant's trust of the survey, which is really the participant's view of the reliability of the survey. Assume that the reliability of the survey depends on - and can be deduced from - the process and conduct of the survey. Then: one, the act of participation would give the participant some insight into the surveying process. I'll assume that the surveying process is honest and is not a deliberate attempt to exaggerate the reliability of the survey. In that case, the participant will either gain no information or a positive amount of information about the process of the survey (i.e. no disinformation).
If the amount of insight is none or negligible, then there should be little or no neglible effect on their trust of the survey.
If they do gain some information about the process (and thus reliability) of the survey, then this information will tend to move the participant's assessment towards the true position: i.e. the actual reliability of the survey.
The second factor, then, is how the participant's a priori assessment of the survey's reliability compares to the actual reliability of the survey. If the participant trusted the survey less than they should (i.e. their subjective, a priori assessment of the survey's reliability is lower than the actual reliability of the survey), then the additional information will move their assessment of reliability will now be higher than before, and they will trust it more.
Conversely, if the participant trusted the survey more than they should (i.e. their subjective, a priori assessment of the survey's reliability is higher than the actual reliability of the survey), then the additional information will move their assessment downward, and they will now trust it less.
When I participated in the Australian Bureau of Statistics's repeated panel data survey of household income, I could see something of the scale, comprehensiveness and relative rigour of the process; at the same time, by participating myself I also gained more knowledge of the weaknesses of the process. Balancing the two, I would say I came to trust the published statistics a little more than before. --PalaceGuard008 (Talk) 05:59, 9 March 2009 (UTC)[reply]
I know that some of the surveys in which I've participated have made me no longer trust any survey results. The would say they "just have a few question", but 100 questions later I was just picking choices at random to make them go away. Then they would ask stupid Q's like "what's your favorite reggae radio station ?", when we don't have any in our area. StuRat (talk) 00:52, 10 March 2009 (UTC)[reply]
The subject of behavioral economics to some extent addresses questions of the sort you are asking. 207.241.239.70 (talk) 16:33, 11 March 2009 (UTC)[reply]

Christians of Bangladesh by division or district

Which divisions or districts of Bangladesh has the most population of Bangladesh?

You mean largest Christian population, I assume? Only about 0.3% of Bangladeshi's are Christians; that's around 1 000 000 overall. Check out the article Christianity in Bangladesh - considering the Portuguese traders settled around Chittagong and Dhaka, I figure those two areas might have the largest Christian diaspora. Master&Expert (Talk) 09:00, 9 March 2009 (UTC)[reply]

I did check the article but it says that the Roman Catholics have dioceses in Dhaka, Rajshahi, Mymensingh, Khulna, Dinajpur and Chittagong. So, I am assuming that these six cities of Bangladesh have Christian population. —Preceding unsigned comment added by 142.204.75.50 (talk) 15:35, 9 March 2009 (UTC)[reply]

food rationing in the US?

Sorry to ask but I'm from Europe and this is a notoriously hidden topic when seen from afar. Is it true what a blogger states today that there are 30 million people in the U.S.A. having their food rationed? --Ayacop (talk) 09:15, 9 March 2009 (UTC)[reply]

I believe the blogger you read was trying to refer to the thirty million Americans who are receiving food stamps.--Rallette (talk) 09:28, 9 March 2009 (UTC)[reply]
Yea, "having their food rationed" makes it sound like there isn't enough food to go around, and people are being forced to eat less. This isn't the case at all. StuRat (talk) 00:43, 10 March 2009 (UTC)[reply]
There are also some food aid programs like WIC that will only pay for certain healthy foods. But that's not really rationing either, If people on WIC want to buy Twinkies and coca-cola they can pay cash.APL (talk) 14:41, 11 March 2009 (UTC)[reply]

Sending flowers to a new baby

Are there any health risks to the mother and newborn if you send congratulatory flowers? Gallaghp (talk) 09:46, 9 March 2009 (UTC)[reply]

There can be, due to allergies, which unfortunately makes this a medical question and probably unsuited for further discussion. Thorough Googling would probably help you out, though; I can't imagine you're the first person to have ever wondered about this.--Fullobeans (talk) 09:58, 9 March 2009 (UTC)[reply]
A query on the possible harmful effects of flowers (or plants) upon postpartum moms and their newborn infants might be posted more successfully on the Science Ref Desk. Note that there, too, actual medical advice is not dispensed. (I'll add from personal experience: In that situation, I most appreciated receiving a decorative, reusable box filled with dried fruits and nuts. The baby appeared oblivious but presumably had no objection.) -- Deborahjay (talk) 13:10, 9 March 2009 (UTC)[reply]
Additionally, the hospital might have some objection to flowers in the maternity wing. I second Deborah's idea of fruit. Dismas|(talk) 13:15, 9 March 2009 (UTC)[reply]
If allergies are a concern with flowers, they may also be with nuts. On the other hand, I think people with nut allergies are only affected if they eat them, and will know not to do it. --Anonymous, 05:08 UTC, March 10, 2009.
Allergies can be strange things. If someone has a severe allergy to nuts, the smell of nuts can be enough to trigger breathing difficulties. Eating nuts is not the only danger; touching surfaces that have been touched by someone who was touching nuts, and then touching their own face or eating something with their hands. With a severe nut allergy, there's a seemingly endless list of ways to go into anaphylactic shock. But if you know the mother well enough to know she probably doesn't have a severe nut allergy, then go for it. If she's sharing a room with someone with a severe allergy, hopefully this should be mentioned when the nuts arrive (sealed in packaging) and she can save them for later. 79.66.56.21 (talk) 07:18, 10 March 2009 (UTC)[reply]

Sending flowers as a congratulatory statement to the new family is nice. I used flowers for corporate gifts. They are "safe" and you need not find detailed information about the likes and dislikes of the recipients. They are also sex netural, depending on the type of bouquet. Flowers are frequently an indulgence a couple would not purchase themselves. Food is necessary for live. Flowers are a luxurious splurge. I purchase ten dollars worth a week to brighten my space. —Preceding unsigned comment added by 75Janice (talkcontribs) 20:35, 11 March 2009 (UTC)[reply]

Sending flowers is nuts. Sending dried fruit is a blooming good idea!! ;-)) Richard Avery (talk) 15:01, 9 March 2009 (UTC)[reply]

Subcamp of Dachau at Königssee vs. Königsee

The Dachau concentration camp was in Bavaria, and I'm trying to determine the location of a particular subcamp ("kommando;" i.e. forced labor camp). A period document relating to Dachau subcamps' staffing, written by one Obersturmführer Wilhelm Ruppert, cites such a camp at Königssee, which is indeed in Bavaria. My problem: The "Catalogue of Camps and Prisons in Germany and German-occupied Territories, Sept. 1st, 1939 – May 8th, 1945" issued in July 1949 by the International Tracing Service HQ at Arolsen (now Bad Arolsen), cites a "CCKdo. of Dachau" at "Koenigsee" (vol. 1, p. 208). It's in the Berchtesgaden kreis, so the location is promising... but per that spelling, we get Königsee and Königsee (Plötzky), the former in Thuringia, the latter in Sachsen-Anhalt. I'm told that the ITS was quite rigorous in its documentation, so how might I reconcile all of these? -- Deborahjay (talk) 13:04, 9 March 2009 (UTC)[reply]

As you probably already expected, Königssee is frequently (mis)spelled Königsee. -- Fullstop (talk) 19:27, 9 March 2009 (UTC)[reply]
Indeed; I've just sent a query via e-mail to the ITS via its website's Contact Us feature. Clarifications will be duly posted on relevant WP pages here and in the German Wikipedia too. -- Deborahjay (talk) 08:24, 10 March 2009 (UTC)[reply]

Another World War II Topic

Its in conection with the 6th Australian Division

In the article its states; During the campaign, Brig. George Vasey's 19th Brigade (minus the 2/11th Battalion) was defeated by the Leibstandarte SS Adolf Hitler brigade, at the Battle of Vevi. The 2/4th and 2/8th Battalions became the only Australian Army units to face elite Waffen SS soldiers in combat.

I am sure this is the only time the Australian Army and the Waffen SS ever met in combat but I am unable to find a source so any help would be appreciated --Jim Sweeney (talk) 14:01, 9 March 2009 (UTC)[reply]

Microsoft Word - Free version of some kind?

I'm an MA History student who's working on his latest essay, and I just finished recovering from my hard drive dying on me. I have a new one, but I now have no Microsoft Word, and I'm currently trying to use Notepad to finish the essay, which is hardly ideal. I downloaded a student version of Microsoft Office called 'The Ultimate Steal', and have sent an email to Microsoft begging for a second chance to download it. Eventually I'll get a copy of Word, one way or another, but for now I'm in something of a bind. Is there a free derivative of it online that can be downloaded (legally, of course) so I can use it as a stop-gap solution. I'm fairly desperate at the moment. Thanks, Skinny87 (talk) 15:15, 9 March 2009 (UTC)[reply]

It's not Microsoft Word, but OpenOffice.org has a free wordprocessor which can read and write Word files. Marnanel (talk) 15:19, 9 March 2009 (UTC)[reply]

open-office is great, I have it myself on the Mac. Other than this though is google-docs (http://docs.google.com/?pli=1#) which allows you to write documents online that you can save and access anywhere. ny156uk (talk) 16:47, 9 March 2009 (UTC)[reply]

If you had a legal copy of Word and it got destroyed when your hard drive crashed then I don't think Microsoft or anyone would object to you finding someone with a CD of Word and installing from their CD. Someone at your college must have one.
OpenOffice is a strange beast (or was when I last used it), and confusing for anyone used to Word. It will probably get you through a crisis. But frankly if this is for one essay then I would recommend using Wordpad. It's many steps up from Notepad, and it will probably take less effort to write the essay with it than to install OpenOffice for doing just one task. DJ Clayworth (talk) 18:08, 9 March 2009 (UTC)[reply]
What country do you live in? That will make a difference to the legality of some methods of acquiring Word. NeonMerlin 20:28, 9 March 2009 (UTC)[reply]
I'm in the United Kingdom, of that helps! Skinny87 (talk) 20:52, 9 March 2009 (UTC)[reply]
OpenOffice is the ticket in the long run. For the short term, Google docs or Zoho writer are a big step up from Notepad and will survive that pending crash. -- Fullstop (talk) 22:44, 9 March 2009 (UTC)[reply]

Please help with Columbine High School Massacre

I am studying at University and I have to categorize the shooters in the scale of evil of American psychiatrist Michael Stone (T.V. program Most Evil), so I read their article because I need to know if they suffered mental illnesses, etc. I found Harris was a psychotic but at the end of the text ... "Harris was a clinical psychopath"... so, was he a psychotic or a psychopath?. And another question ... if you are a depressive person (Klebold), are you a psychotic or a psychopath?.. thanks a lot!!!! It's University homework but I have attempted an answer first and don't understand that. --190.49.101.52 (talk) 15:48, 9 March 2009 (UTC)[reply]

It's hard enough to diagnose mental illnesses when interviewing a living subject and giving them a battery of psychological tests. It's harder still when dealing with psychopaths, as they're so good at concealing their inner selves. Surely it's impossible to make any worthwhile diagnosis of a dead person? Mimetic Polyalloy (talk) 16:23, 9 March 2009 (UTC)[reply]
If you are writing for a University, don't forget to check whether they consider Wikipedia an appropriate source. Many don't, in which case you'll have to find evidence elsewhere. If you find any please come back and add to the article. DJ Clayworth (talk) 18:01, 9 March 2009 (UTC)[reply]
Actually, there is nothing wrong with reading Wikipedia to get information. You, of course, would never cite Wikipedia in any scholarly paper, but that has nothing to do with the quality of Wikipedia as an encyclopedia. You wouldn't ever cite it in a scholarly paper because it is an encyclopedia. Instead, what you should do is read Wikipedia to get an overview on the topic, and then follow the references to find sources you WOULD cite. Well written articles have sources which themselves can be read, and THOSE can be cited... --Jayron32.talk.contribs 18:26, 9 March 2009 (UTC)[reply]
I guess the point I was trying to sneak up on is this. Wikipedia contains information which is contradictory. That's actually pretty common. Someone reads an article saying that Harris was psychotic, so they add that to the article. Someone else reads one saying that Harris was a psychopath, so they add that to the article. Very rarely does anyone do the (extremely hard) work of researching to find out whether these are contradictory or if they are which one is the consensus, or if they are both educated guesses. DJ Clayworth (talk) 15:14, 10 March 2009 (UTC)[reply]
As you don't seem to have received an answer, if I reach back 30 years in my memory I find that depression is usually classified as neurosis, whereas psychosis refers to conditions with hallucinations or delusions. Psychopathology often refers to conditions which are incurable and/or untreatable, whereas psychoses/neuroses respond to treatment. Our article on psychosis refers to "In contemporary culture, the term "psychotic" is often incorrectly used interchangeably with "psychopathic or sociopathic", which actually describe a propensity to engage in violently antisocial behaviors, not usually involving hallucinations or delusions." Hope this helps. --TammyMoet (talk) 18:45, 9 March 2009 (UTC)[reply]
You say you're studying at a university where you're asked to rate the evil of the Columbine shooters on a scale as presented by a tv program, Most Evil?--Wetman (talk) 02:53, 10 March 2009 (UTC)[reply]
...in which case I'm guessing using Wikipedia as a reference is going to be perfectly acceptable. DJ Clayworth (talk) 15:14, 10 March 2009 (UTC)[reply]
The OP doesn't specify what subject they are studying... Presumably it isn't psychology! --Tango (talk) 15:24, 10 March 2009 (UTC)[reply]
I am not sure if this helps, but: There actually is an article on Most Evil and the scale of evil deployed by Michael Stone. Categories 01 to 22 are listed together with the names of the criminals profiled in this TV "show". --Cookatoo.ergo.ZooM (talk) 17:10, 10 March 2009 (UTC)[reply]

Desirable self-fulfilling prophecies

The examples given at Self-fulfilling prophecy are almost all undesirable. What desirable effects can be achieved by self-fulfilling prophecies? NeonMerlin 20:24, 9 March 2009 (UTC)[reply]

Multiple investors could see a low stock as vastly undervalued, which leads to them buying it, which leads to the price of the stock rising, which leads to wealthier investors and the stock (with its new inflated price) showing how undervalued it was. Livewireo (talk) 20:54, 9 March 2009 (UTC)[reply]
Ah, whatever you like? An example: let's say that the prophecy is that you're going to become a rich and successful man by inventing a new computer operating system. Your self-confidence and motivation are boosted by the information that your labors will not be in vain, and naturally you take an interest in programming, since that's where your big success is supposed to come from. Consequently, you create MerlinOS, which sells like hotcakes and becomes a new industry standard, and you indeed become a rich and successful man. If it wasn't for the prophecy, you would have gone on to become, uh, I don't know. A chimney sweep! -- Captain Disdain (talk) 21:06, 9 March 2009 (UTC)[reply]
There are two bullet points at at Self-fulfilling_prophecy#Real-life_examples. The second one has examples of desirable outcome. -- Fullstop (talk) 22:30, 9 March 2009 (UTC)[reply]
Is successful cognitive behavioral therapy a form of self-fulfilling prophecy? --Tagishsimon (talk) 23:47, 9 March 2009 (UTC)[reply]
If we're talking about mythology specifically, there's a very good reason why almost all of them have bad outcomes. The lesson those stories are trying to teach is that you don't mess with fate. The stories all follow the same basic structure: first, someone receives a prophecy (it could be in a dream, or from an oracle or something). Then they foolishly make every attempt to stop the prophecy from coming true. And finally, the prophecy comes true because of the steps taken to avoid it.
What's the lesson here? It's this: don't mess with fate!!! Don't do it! Your fate is your fate, and it's hubris to try and avoid it. You can't do anything about it, and if you try, it's just going to get even worse in the end.
One should note that it isn't universal in mythology that the outcome is bad (although it's vastly more common). The article mentions a few fairy tales where the outcome is good, but almost all of them follow that same structure too, the difference being that it's usually the bad guys trying to stop the (good) prophecy from coming true instead of the other way around, and they are the ones to pay the price. Belisarius (talk) 23:51, 9 March 2009 (UTC)[reply]
Moore's law. Applications engineers count on the projected abilities of the hardware that will be available when their project goes to market. Chip-makers, in turn, know that there will be a market for their improved chips, without which they wouldn't invest in improving the technology. —Tamfang (talk) 04:15, 10 March 2009 (UTC)[reply]

really 12-year low?

The dow is what it was 12 years ago. But doesn't that mean that it's really much, much lower, because isn't a 2009 dollar worth WAY less than a 1997 dollar? And isn't the dow denominated in current dollars? Or am I missing something... —Preceding unsigned comment added by 85.181.144.221 (talk) 23:38, 9 March 2009 (UTC)[reply]

I don't think you are missing anything. But I think we're all far too depressed to begin calculating the inflation adjusted matching point. --Tagishsimon (talk) 23:45, 9 March 2009 (UTC)[reply]
Math is simply a luxury we can't afford in these troubled times. Belisarius (talk) 23:58, 9 March 2009 (UTC)[reply]
Using the GDP deflator to compare prices from the different dates, you'd have to go back to November 15, 1995, to get a closing quote for the Dow as low as today's, in inflation-adjusted terms. So, in real terms, today's Dow close was a 13-year, 4-month low. Stocks rose rapidly during the mid-1990s, so you don't have to go much farther back in time from the nominal price to get the same real price. Marco polo (talk) 00:55, 10 March 2009 (UTC)[reply]
The Dow isn't "denominated in dollars", it's a scaled average. See Dow Jones Industrial Average. --LarryMac | Talk 14:04, 10 March 2009 (UTC)[reply]
Agreed and it is helpful to note this, since it was one of the OPs questions: but probably equally important to state that it has no bearing on the other element, which is that the index does not adjust for inflation. --Tagishsimon (talk) 14:21, 10 March 2009 (UTC)[reply]
Yes, the scaling is just to account for non-market changes to the stock price (like stock splits). Zain Ebrahim (talk) 14:30, 10 March 2009 (UTC)[reply]
True enough, but the components of the Dow also have changed, making any comparison to the index value from twelve or more years ago somewhat suspect as well. --LarryMac | Talk 14:37, 10 March 2009 (UTC)[reply]
However, it is a scaled average of prices that are denominated in dollars, so the original poster's point about inflation is correct. (That is, it's correct to the extent that values in different years can be compared; see Larry's point about the components changing.) --Anonymous, 19:44 UTC, March 10, 2009.


March 10

Louis H. Sullivan's Charnley House Architecture Influence from Germany

I saw a large Architecture coffee table book a few years ago, that had a photo of the Charnley house, and then a description of German Residential architecture in a upper middle class Berlin Neighborhood in 1860's, that influenced the Charnely House. Does anyone know what area that was? Thanks for any help —Preceding unsigned comment added by Srader (talkcontribs) 01:19, 10 March 2009 (UTC)[reply]

Arabs in Bangladesh

Are Palestinians the only Arabs in Bangladesh? —Preceding unsigned comment added by 74.14.119.105 (talk) 02:03, 10 March 2009 (UTC)[reply]

What nationality?

I'm looking through a set of photos on the Telegraph website, about an increase in security around Tibet. Now, this photo seems to clearly say "Nepal Police", despite the captioning. Based on that, this also seems to be a photo of Nepali police officers. Could experts give an opinion on which of these photographs actually show Tibet (as opposed to Nepal or India)? --PalaceGuard008 (Talk) 08:08, 10 March 2009 (UTC)[reply]

If it helps at all, the Alt-text of both images refers to Tibetan Buddhists and Nepalese police in Kathmandu --Saalstin (talk) 14:04, 10 March 2009 (UTC)[reply]
Further alt-text checking yields the following locations for each photo number: China: 1, Kathmandu: 2,5,6,7,8,9,12, Dharamsala: 3,9,11,13,14, Tibet: 10,15,16,17 --Saalstin (talk) 14:14, 10 March 2009 (UTC)[reply]
There have been several protests in Nepal for Tibetan independence, most of which have been dealt with harshly. So basically, the images are of Nepalese police, in Nepal, but are part of the same clampdowns on protesters as in China. - Jarry1250 (t, c) 16:48, 10 March 2009 (UTC)[reply]
The Telegraph (UK) says:
1: "A Tibetan Buddhist prostrates himself before the Bouddhanath stupa",
2: "Nepalese policemen stand guard as Tibetan Buddhist monks attend Kora prayers at Bauddhanath Stupa in Kathmandu". --Cookatoo.ergo.ZooM (talk) 16:56, 10 March 2009 (UTC)[reply]
Thanks. I should've been looking at the alt text, not the textual caption. --PalaceGuard008 (Talk) 21:34, 10 March 2009 (UTC)[reply]

was america the first modern democracy?

was America the first modern (17th-18th c.) country to develop as a democracy? or did it follow democracies elsewhere (where?) Thanks. —Preceding unsigned comment added by 92.230.64.67 (talk) 10:03, 10 March 2009 (UTC)[reply]

Democracy#Middle Ages chandler · 10:13, 10 March 2009 (UTC)[reply]
See also American Exceptionalism. Malcolm XIV (talk) 10:27, 10 March 2009 (UTC)[reply]
History of democracy is probably a better link.
The path to democracy was a long slow one, from the oligarchies of Venice to the 18th and 19th century democracies which restricted suffrage by race, gender, or wealth, to the present day, so while the USA has a claim, it's hard to say which was the first modern democracy; the United Kingdom (England and Scotland before 1707) had a functioning parliament since the 17th century, but not everyone could vote. The short-lived Corsican Republic founded in 1755 has a strong claim to precede the USA. --Maltelauridsbrigge (talk) 10:47, 10 March 2009 (UTC)[reply]
Actually, England's first elected parliament was in the 13th century. And it's worth bearing in mind that "not everyone could vote" also applies to the United States until the 20th century. Malcolm XIV (talk) 11:00, 10 March 2009 (UTC)[reply]

maybe you guys missed it but I said first MODERN democracy, and I meant 1600s-1700s not the middle ages. By this standard it seems the anwer is "yes" (for example your "to the 18th and 19th century democracies" refers, first, to America, doesn't it? —Preceding unsigned comment added by 92.230.64.67 (talk) 11:49, 10 March 2009 (UTC)[reply]

If you constrain the terms "modern" and "democracy" sufficiently enough, you can make a case for America being the first modern democracy. But its an "ex-post-facto" distinction, created to give substance to American exceptionalism. With the exception of universal adult sufferage (which America was NOT the first nation to enact, Norway had America beat by 7 years) all of the aspects of the American Democracy existed in other nations earlier than America was formed, often by centuries. As noted, the idea of the elected assembly existed in Europe for hundreds of years before the Europeans even knew about the Western Hemisphere, Parliament first met in the 13th century, and the French Estates-General met for the first time in 1302. Elected executives existed in the Dutch Republic in the 1580's (Stadtholder), and federalism in the same, as well as in Switzerland from 1291. Nearly every aspect of the American system existed elsewhere earlier. One might be able to make a case that it was the first country with an elected executive AND elected legislature AND universal adult sufferage all at once, but that only happened in 1920, and other countries (which later reverted to dictatorships) would meet all three criteria a few years earlier. --Jayron32.talk.contribs 12:31, 10 March 2009 (UTC)[reply]
"First modern" makes no sense unless there was significant gap during which there were no democracies (which I don't believe there was), otherwise the first modern democracy is just the first one to occur during the the period you define as being "modern". --Tango (talk) 13:40, 10 March 2009 (UTC)[reply]
I see where you're going with this. But during the middle ages there WAS a gap, it seems, at least to read the responses. Or are there 1400s and 1500s democracies America was fashioned after? —Preceding unsigned comment added by 92.230.64.67 (talk) 14:01, 10 March 2009 (UTC)[reply]
There is no gap in the history of democracy in, for example, the United Kingdom, the Venetian Republic or the Dutch Republic between the Middle Ages and the "Modern" period. If you genuinely want an informative answer, please read the articles referenced above; if you just want an argument, take it elsewhere. This is not the place to start discussions or debates. ˜˜˜˜ —Preceding unsigned comment added by 209.251.196.62 (talk) 14:16, 10 March 2009 (UTC)[reply]
At the time of the American War of Independence Britain had a well established functioning parliament. "No taxation without representation" was not a complaint that nobody had representation, just that the American colonies didn't have enough. DJ Clayworth (talk) 15:01, 10 March 2009 (UTC)[reply]
Did the American colonies have any representation in parliament? A Quest For Knowledge (talk) 16:08, 10 March 2009 (UTC)[reply]
Not directly, I don't think. Whether they needed direct representation in order to have some form of representation was a topic of active debate at the time. Most Americans obviously believed they did; Parliament itself seemed to be on the fence about it. --140.247.251.34 (talk) 16:13, 10 March 2009 (UTC)[reply]
No Taxation without Representation mentions virtual representation. Our article actually tells people to "google search 'declatory act imperial representation' " Jeez.. A Quest For Knowledge (talk) 16:17, 10 March 2009 (UTC)[reply]
The recent democratic experiment that was in the forefront of American consciousness in the 1770s was that of Pasquale Paoli in Corsica.--Wetman (talk) 14:59, 10 March 2009 (UTC)[reply]

I'm surprised no one has mentioned female suffrage (in case that was the 1920 reference). It seems that no definition of 'modern' democracy could stand without that. The USA was of course, behind NZ and doubtless a few others..81.140.37.58 (talk) 17:14, 10 March 2009 (UTC)[reply]

Short answer to the original question: No. None of the democracy articles mention the Tynwald, possibly because elections only began in 1866. You could argue that the US is still not a democracy, as many people are barred from voting. 89.242.99.255 (talk) 17:17, 10 March 2009 (UTC)[reply]

Barred from voting in the US? Like who? Children, felons and those too lazy to go vote? 12.216.168.198 (talk) 17:42, 10 March 2009 (UTC)[reply]
I've heard that anyone convicted of a crime in the US is banned for life from voting, which I understand in some neighbourhoods makes up a large portion of the population. On this side of the pond, criminals are only barred from voting while in prison - once released they can vote again. 78.146.23.195 (talk) 01:29, 11 March 2009 (UTC)[reply]
It really varies quite a bit between the different states, and misdemeanors usually have no effect... AnonMoos (talk) 01:34, 11 March 2009 (UTC)[reply]
Non-citizens, too. (Some of them, anyway - I'm not sure about the US, can non-citizen residents vote at all? Local elections, maybe?) --Tango (talk) 17:46, 10 March 2009 (UTC)[reply]


Kpalion(talk) 18:07, 10 March 2009 (UTC)[reply]

I have trouble with the term "universal adult suffrage", because the term "adult" is itself widely open to interpretation. Is that 21 ? Or 18 ? Or something else ? In former times, people much younger than that were able to marry and form households. And how about those outside the normal mental aging process ? Can the mentally retarded be considered to never be "mental adults", and thus barred from voting, at any age ? And does a genius who obtains a college degree at age 12 also deserve the right to vote then ? StuRat (talk) 18:38, 10 March 2009 (UTC)[reply]
I think what the phrase really means is "suffrage not dependant on race, gender or wealth" - plenty of other factors are considered, so it is rarely completely universal. It's not a good term... --Tango (talk) 18:45, 10 March 2009 (UTC)[reply]
The U.S. didn't actually have fairly uniform unqualified white male adult suffrage across the various different states until Andrew Jackson's time (or until after the Dorr Rebellion in the case of Rhode Island). AnonMoos (talk) 19:12, 10 March 2009 (UTC)[reply]

I'm not sure what "modern democracy" particularly means, but the U.S. was the first geographically large-scale democracy (i.e. whose electoral base was not confined to a single city-state or a small geographic area) whose electorate also extended significantly beyond a relatively small economic elite. Obviously it built on the then limited example of British parliamentarism, as well as the colonial experience in North America, with some allusions and influences from the early Roman Republic and ancient Athens. AnonMoos (talk) 19:00, 10 March 2009 (UTC)[reply]

You can't choose to limit democracy by excluding those countries or states whose geographic size happens ro be smaller than America. As others have said don't try to limit the definition to fit the answer you want. Tmol42 (talk) 19:14, 10 March 2009 (UTC)[reply]
Dude, what are you angry about? —Ed 17 (Talk / Contribs) 19:20, 10 March 2009 (UTC)[reply]
Whatever, dude -- I don't particularly understand the original question, and I frankly doubt whether the original question really has any very precise meaning to be understood, so I chose to limit the parameters to something which actually has a definite answer. And I didn't say anything about states "smaller than America"[sic] -- I was referring to entities much larger than a city-state, a Swiss canton, or a Netherlandish province... AnonMoos (talk) 19:24, 10 March 2009 (UTC)[reply]
Of course you bar non-citizens from voting. Otherwise, what is to stop the US from sending 10 million people to Alberta during elections and vote to secede from Canada and join the US so that the US gets all that oil? 65.167.146.130 (talk) 19:28, 10 March 2009 (UTC)[reply]
First, provinces don't have the right to secede any more than US states do. Which is not to say that it would not be allowed in the presence of bona fide popular support, as Sweden did with Norway.
But as long as non-residents are not allowed to vote, immigration controls would block any such strategy.
Since we're talking about Canada, I should also note that non-citizens were allowed to vote until 1975 if they were Canadian residents who were "British subjects", which basically means citizens of any place in the British Commonwealth. (That's more a measure of the gradualness of Canadian independence than anything else. Canadian citizenship didn't even exist as a distinct category until 1947.)
--Anonymous, 17:00 UTC, March 10, 2009.
I'm a resident in New Zealand, but not a citizen. I regularly vote in elections here. The idea of prohibiting residents from voting seems very strange and undemocratic to me.-gadfium 20:29, 10 March 2009 (UTC)[reply]
In many parts of the U.S. during much of the 19th century, foreigners who were permanent residents (and so presumably on the path to naturalization) were allowed to vote in local and municipal elections (but not in statewide or federal elections). This was elminated largely towards the end of the 19th-century as part of the wave of reforms aimed at diminishing the power of city political bosses (and also because at that time recent immigrants were skewing heavily towards eastern and southern Europe). It was one of several "Progressive" reforms of the period which don't necessarily always seem truly progressive with modern hindsight... AnonMoos (talk) 04:40, 11 March 2009 (UTC)[reply]

It's worth noting here, perhaps, that the Founding Fathers of the United States didn't really think in terms of founding a democracy; they sought to create a republic. Advocates of republicanism worried that government by democracy might become a "mobocracy" or a tyranny of the majority. It's been argued that Thomas Paine was the only Founder who advocated democracy in the modern sense of the term. But by finally getting rid of lords and kings and other medieval leftovers of a deferential society guided by hereditary rule--a process already begun in Britain--the Founders created the conditions for democracy to emerge. —Kevin Myers 21:41, 10 March 2009 (UTC)[reply]

We'll have to work hard to define the words "modern", "country" and "democracy" to allow the US to be the first. They were slow to offer universal suffrage, for a start. And, as pointed out above, they still restrict voting to citizens, where most democracies allow residents to vote. They were not the first modern republic, and their democratic method was based on the systems already in place in Europe. They did not invent democracy, nor were they the first to implement it. However, the US were the first to introduce the US constitution. Gwinva (talk) 22:12, 10 March 2009 (UTC)[reply]
  • "Most democracies allow residents [rather than only citizens] to vote" is not what it says at Right of foreigners to vote, although that article could use a summary section with some actual statistics. --Anonymous, 02:36 UTC, March 12, 2009.
Well, no shit... But they were not the first to introduce a "constitution" in the sense of a single governing document that outlined the organization and role of the government. The Constitution of San Marino was laid out in 1600. The Zaporizian Cossacks wrote one in 1710, though it failed to go into effect when they failed in their bid for an independant Ukrainian state. Corsica had one in 1755, which established a fully republican form of government. Once again, the U.S. wasn't the first, though we certainly brag like we are. --Jayron32.talk.contribs 22:23, 10 March 2009 (UTC)[reply]
Sorry, sarcasm doesn't work well in text form. I was merely trying to show that to make the US the "first" we'll have to narrow the definition down until it applies, well, only to the US. Gwinva (talk) 01:33, 11 March 2009 (UTC)[reply]
Doh! Sorry about that! --Jayron32.talk.contribs 04:52, 11 March 2009 (UTC)[reply]

Lots of monarchies (which had some sort of assembly) and republics are listed as shining examples of democracy preceding the U.S. What percentage of the U.S. citizens were eligible to vote in 1789 (considering lack of women's suffrage, and the exclusion of any other categories, such as slaves, felons and those failing literacy tests or not paying a poll tax? By the same measure, what percentage of British subjects in the UK were eligible to vote? Same question applies to other predecessor democracies at the founding of the U.S. under its then and present Constitution? The article Parliament of Great Britain says "Candidates for the House of Commons stood as Whigs or Tories, but once elected formed shifting coalitions of interests rather than splitting along party lines. At general elections the vote was restricted to property owners, in constituencies which were out of date and did not reflect the growing importance of manufacturing towns or shifts of population, so that in rotten boroughs seats could be bought or were controlled by rich landowners, while major cities remained unrepresented." See also Rotten and pocket boroughs and Old Sarum (UK Parliament constituency), an uninhabited hill which belonged to the Pitt family, and whose owner was entitled to send two representatives to the House of Commons until 1832. The recognized right to openly buying the right to a seat in the national assembly was not a good exemplar of democracy. Edison (talk) 22:51, 10 March 2009 (UTC)[reply]

Britain was a relatively shining example of a persistent representative form of government able (after 1689) to impose certain limits on arbitrary executive power, but before 1832 the whole representative basis of parliament was extremely erratic and slipshod, and until 1867 and later, very few people would have claimed that Britain was already a democracy, and rather few people among the political elites even thought that democracy was a good thing... AnonMoos (talk) 01:16, 11 March 2009 (UTC)[reply]

There is some conventional wisdom (or maybe it's just a cliché in some circles) saying that the US is the world's second-oldest democracy, with Switzerland as #1. 207.241.239.70 (talk) 03:09, 11 March 2009 (UTC)[reply]

Some might say that there was no real Swiss national government (democratic or undemocratic) until 1848... AnonMoos (talk) 04:40, 11 March 2009 (UTC)[reply]

First female dentist

Who was the first female dentist? --85.226.44.201 (talk) 11:41, 10 March 2009 (UTC)[reply]

In America, Lucy Hobbs Taylor. Zain Ebrahim (talk) 12:47, 10 March 2009 (UTC)[reply]
And Europe? Perhaps Amalia Assur?--85.226.44.201 (talk) 13:52, 10 March 2009 (UTC)[reply]

why would blondes be "dumb"?

I don't get where the stereotype comes from, any ideas? —Preceding unsigned comment added by 92.230.64.67 (talk) 12:45, 10 March 2009 (UTC)[reply]

I think it's probably more that "dumb" people dye their hair blonde - blonde is, for some reason, considered prettier than other colours and "dumb" people are more likely that other people to care about fitting into pretty stereotypes. "Dumb" in this context tends to mean "naive" more than anything. --Tango (talk) 13:37, 10 March 2009 (UTC)[reply]
I don't really think the dumb blonde stereotype has anything to do with dying hair. Our article Dumb blonde offers some information on it, but doesn't really get into its origins. Tomdobb (talk) 15:29, 10 March 2009 (UTC)[reply]
In modern culture, the "blonde bimbo" stereotype was exploited by Hollywood in the 50s (if not earlier). Once you start a stereotype, selection bias will keep it going. No matter how many smart blondes or dumb brunettes a person meets, only the dumb blondes are noticed, which continues the stereotype. -- kainaw 15:34, 10 March 2009 (UTC)[reply]
I agree with the hair dye cause. Another factor is that very few people in the US (where I believe this stereotype originated) have naturally blond hair. Therefore, of all those people who have blond hair, a large portion have dyed it. If people who dye their hair are more concerned with appearance than developing their intelligence, then this could, indeed, mean that the average blond has a less developed intelligence. This would not, of course, apply to that portion of the blond population which is naturally blond. Another factor is that some of those women who dye their hair blond may actually be intelligent, but may choose to act stupid, as this goes along with the image of themselves they are trying to project. And, finally, some intelligent, naturally blond women may opt to dye their hair darker, if they feel this is necessary to overcome the stereotype of having people think of them as stupid. This would further deplete the blond population of intelligent specimens. StuRat (talk) 18:28, 10 March 2009 (UTC)[reply]
After the fun I had writing Long hair, I'm tempted to try a hand at the Dumb blonde article. Wrad (talk) 18:38, 10 March 2009 (UTC)[reply]
After a little bit of research, the best I could come up with was that the modern stereotype seems to stem from the book Gentlemen Prefer Blondes and its subsequent movie version. Tomdobb (talk) 18:45, 10 March 2009 (UTC)[reply]
That would have been my guess. The modern stereotype seems to have been popularized by Marilyn Monroe, who was probably influenced by the dimwitted showgirl type played so well by Judy Holliday. Gracie Allen probably helped established the stereotype too. —Kevin Myers 21:08, 10 March 2009 (UTC)[reply]
I would be extremely interested to see some numbers or research to back up this "hair dye" theory. I don't agree with it at all, but I do see Kainaw's explanantion as more valid. 10draftsdeep (talk) 21:29, 10 March 2009 (UTC)[reply]
The "dumb blond" stereotype was in newspapers by 1927, when the Chicago Tribune said " A large beautifu and dumb blonde was visiting on the set." In 1938 the Chicago Tribune said "the day of the dumb blonde is done" The Los Angeles Times in 1941 said in a story "Mr. DWG and the B. but D. Blonde, "A large beautiful and dumb blond was visiting on the set." The Los Angeles Times in 1941 said"The strictly Dumb Blonde days are slipping into history." There are a number of cites from the 1920's and 1930's referring to the stereotype. Edison (talk) 22:37, 10 March 2009 (UTC)[reply]
The hair dye theory is mostly anecdotal - most dumb blondes I've met have been dyed blondes. --Tango (talk) 00:01, 11 March 2009 (UTC)[reply]
That sounds like OR to me, but since I live with a very intelligent natural blonde I guess I would be guilty as well if I stated my experience. cheers, 10draftsdeep (talk) 00:47, 11 March 2009 (UTC)[reply]

Even if a person's hair is naturally blond when she is young, doesn't it typically turn brown by the time she is an adult, requiring chemical assistance to keep it as "blonde" as it was in the teen years? Edison (talk) 15:36, 11 March 2009 (UTC)[reply]

Sometimes, not always. Some adults are naturally blonde. --Tango (talk) 16:31, 11 March 2009 (UTC)[reply]

Dow Jones

I've just been reading Dow Jones Industrial Average#Criticism, and it seems to me that the Dow Jones is an almost completely meaningless measure of the market. A stock split should never be able to affect an index, since it's nothing more than an accounting fudge to make things more convenient, yet it will affect the Dow Jones Industrial Average - the scaling is adjusted to prevent any immeadiate effect, but from then on the DJIA will have a different value than it would have done without the split. To give an extreme example, assume two stocks, each worth $1/share with the scale factor set at 2, so the DJIA is 1. Now one of those shares has a 100:1 split, making it's price $0.01/share, in order to keep the DJIA at 1 the scale factor is adjusted to 1.01. Now the price of the other stock goes up to $2 (while the other stays unchanged), the DJIA is now 1.99. Had the stock split not happened the DJIA would have been 1.5. Am I misunderstanding something, or is the DJIA really such a meaningless number? --Tango (talk) 14:55, 10 March 2009 (UTC)[reply]

I don't quite get the scaling factor of 1.01. Can you explain that ? Here is what I would expect would happen both ways:
WITHOUT THE SPLIT/BEFORE RISE | WITH THE SPLIT/BEFORE RISE
----------------------------- | ---------------------------
Stock A = $1.00 × 0.5 = 0.5   | Stock A = $1.00 ×  0.5 = 0.5
Stock B = $1.00 × 0.5 = 0.5   | Stock B = $0.01 × 50.0 = 0.5
                        ---   |                          ---
                        1.0   |                          1.0
                              |
WITHOUT THE SPLIT/AFTER RISE  | WITH THE SPLIT/AFTER RISE
----------------------------  | --------------------------
Stock A = $2.00 × 0.5 = 1.0   | Stock A = $2.00 ×  0.5 = 1.0
Stock B = $1.00 × 0.5 = 0.5   | Stock B = $0.01 × 50.0 = 0.5
                        ---   |                          ---
                        1.5   |                          1.5
So, either way, the DOW would go up 50%. Perhaps you are thinking they scale the entire DOW up or down, due to a split, when I would assume they only change the weighting factor on that particular stock. See weighted average. StuRat (talk) 18:07, 10 March 2009 (UTC)[reply]
If I'm interpreting Dow Jones Industrial Average#Calculation correctly, they really do scale the whole thing. The average isn't weighted. That's what seems to make it meaningless. --Tango (talk) 18:13, 10 March 2009 (UTC)[reply]
Your calculation is correct. But note that it is quite an extreme example. The DJIA contains 30 large companies and you won't expect their stock price to double very often. But it does show that using price-weighted indices to establish trends are not very useful in volatile times. Some other drawbacks are the unrepresentative sample and additional weight given to high-priced stocks. But if you assume that people who trade in this index understand its divisor fully (I believe they do) then it is useful (to them) - it's useful to use to bet on the companies in it. But using it to measure the health of the economy is wrong. Zain Ebrahim (talk) 18:44, 10 March 2009 (UTC)[reply]
Yet many people use it for that purpose. It's commonly reported as the standard measure of what is going on in the US markets. --Tango (talk) 23:46, 10 March 2009 (UTC)[reply]
While it's reasonable to use it to show short-term changes in the market (say over the last year), it's not nearly as good over the long term (say 50 years). StuRat (talk) 15:09, 11 March 2009 (UTC)[reply]
Actually, since it is easy to keep track of the changes to the divisor, I have seen studies that showed trends of the DJIA after adjusting for any changes - but not anywhere near 50 years. It's normally included for interest, in addition to one of the other more diversified indices. Also, it is useful to know how the DJIA performed on a certain day/week for an idea of the performance of the large companies on that day/week. Zain Ebrahim (talk) 18:13, 11 March 2009 (UTC)[reply]

It matters less whether a measure is accurate, than whether it is consistent. The measurement may be flawed, but if the flaw is consistent then the changes in that measurement can be trusthworthy. 194.221.133.226 (talk) —Preceding undated comment added 15:15, 11 March 2009 (UTC).[reply]

But it isn't consistent - a minor accounting change can change the measurement. It isn't that the data isn't trustworthy, it's just that it is meaningless. The average all the per share prices, scaled by a certain amount definitely is that number, but who cares? --Tango (talk) 16:27, 11 March 2009 (UTC)[reply]
Just becuase it is less than 100% logically consistent, it doesn't mean it is meaningless. See fuzzy logic, for example.
The US seems to be very good at patching up something anachronistic which really doesn't work very well and keeping it spinning for *years*. Like the whole floor trading business, for example. And Imperial measures. --PalaceGuard008 (Talk) 21:39, 11 March 2009 (UTC)[reply]

IMMIGRENT VISA

WHY IS A IMMIGRENT VISA CALLED A GREEN CARD WHEN IT IS ACTUALLY YELLOW IN COLOR —Preceding unsigned comment added by BIGDADDYSCHUCKWAGON (talkcontribs) 15:48, 10 March 2009 (UTC)[reply]

This is a violation of WP:NONSENSE AltecLansing12 (talk) 15:49, 10 March 2009 (UTC)[reply]

Not exactly sure why you'd call it a violation of nonsense...Anyway, it is called a green card because the original United States Permanent Resident Card was green. --OnoremDil 15:54, 10 March 2009 (UTC)[reply]
Agreed (on both points). There are many instances where an item is known by it's former color, such as the pink slip for a car, or the Pontiac Silverdome, which now has a white dome. StuRat (talk) 17:46, 10 March 2009 (UTC)[reply]
And some things are named for colours they've never had; white wine has always been yellow. NeonMerlin 04:27, 12 March 2009 (UTC)[reply]
And it's made from white grapes, which have always been green. StuRat (talk) 06:43, 13 March 2009 (UTC)[reply]
Eh? Mine's greenish - with tons of green writing and barcodes on the back-side. (See image)

I remember a time when there weren't Yellow Pages in the phone book but Pink Pages. Pity they didn't keep the original name when they changed the colour (the way they kept the original name of the Women's Weekly when it became a monthly publication). It would have spawned regular questions of this nature. -- JackofOz (talk) 19:25, 16 March 2009 (UTC)[reply]

Monroe Doctrine entry

I removed several instances of vandalism in this entry. If someone could possibly check my work I'd be grateful! —Preceding unsigned comment added by Sylviably (talkcontribs) 15:50, 10 March 2009 (UTC)[reply]

It looks like you did good, thanks for your help! --Tango (talk) 16:05, 10 March 2009 (UTC)[reply]
Thanks for the info, but this is not the place to ask things like this. You should post a message on the talk page of the article in question. DJ Clayworth (talk) 17:22, 10 March 2009 (UTC)[reply]

Personality development

Reposted from the Language Ref Desk
what are the investigate factors which contibute to personality devolepment?--Munnusmail (talk) 13:03, 10 March 2009 (UTC)[reply]

This sounds like a homework question to me - could you explain why you are asking? --Tango (talk) 16:20, 10 March 2009 (UTC)[reply]
The article on personality development has some details and links to Sigmund Freud, Jean Piaget and others. --Cookatoo.ergo.ZooM (talk) 16:43, 10 March 2009 (UTC)[reply]

paying off my credit card (or not..)

Hello, I've been trawling the wiki reference desk to procastinate and the question on Credit card interest rates got me thinking... Basically, it seems that as someone who is in rented accommodation, without a job (yet!) or the possibilty of entering the housing market any time soon (5 years min...), can i just not pay off my credit card bill? I'm in the UK so would just like to know what the consequences would be.. Many thanks, 81.140.37.58 (talk) 17:18, 10 March 2009 (UTC)[reply]

I don't think we have enough information to help you, since we do not know what other choices you have. Paying off the credit card, if you are indebted, sounds to be a good thing in that you'll be charged less interest in the future. Indeed, that, and less cash in hand, would seem to be the major consequences of paying it off. --Tagishsimon (talk) 17:23, 10 March 2009 (UTC)[reply]
I agree, if you have enough cash to do so, pay it all off as soon as possible. You will be saving a lot of money from not being charged interest. If on the other hand you mean not make any payments, then that is a bad idea. If you do that then you would not be able to get a loan for a car or a mortgage when you get a job. Besides that, its dishonest. And more practically, you will still end up paying it, you will just have to pay the court fees in addition, and more interest. 78.146.23.195 (talk) 01:41, 11 March 2009 (UTC)[reply]
If you don't pay at least the minimum each month you risk bankruptcy, which is very bad. It's not just mortgages you won't be able to get - you won't be able to get a contract mobile phone, you'll have to pay for electricity/gas/water/etc. upfront (with a pre-paid meter), etc. You don't want that unless it really is your only option. If you do pay just the minimum each month it will take you forever (give or take!) to pay it off and the interest you will pay will be enormous (far greater than the initial cost of whatever you bought on the card). If you are in financial difficulties, seek professional advice - not paying your credit card bill is not a good option. --Tango (talk) 17:32, 10 March 2009 (UTC)[reply]

That was really good advice -Tango sounds just like my dad (in a good way...)! Believe it or not, i never though bankrupcy was rally that bad. But yes, i'll be a good boy from now on...81.140.37.58 (talk) 17:36, 10 March 2009 (UTC)[reply]

Unfortunately, you are not alone in thinking that, which is one of the reasons bankruptcy rates are so high (the fact that we're in a recession doesn't help, of course! But the rates were high before that.) --Tango (talk) 17:50, 10 March 2009 (UTC)[reply]
Even if you are not in financial difficulties paying off your credit card is always a good idea. The interest rates charged by credit cards are some of the highest around, which makes it the most expensive way to borrow money. Always pay off the most you possibly can, and if you find you are running a balance for any time consider taking out a lower interest rate loan to pay it off (unless that might tempt you to run up even more debt, in which case take out the lower interest loan and cut up the credit card). DJ Clayworth (talk) 20:15, 10 March 2009 (UTC)[reply]
The best place to go for advice is an accountant or lawyer. If you can't afford that (I guess you can't) then the Citizens Advice Bureau is the place to go (unfortunately they're swamped right now, so get on the list for an appointment before you're in dire straights). Beware of insolvency practitioners you see advertised; some are professional and have your best interests at heart, others want to rush you into an IVA, take their fee, and leave you with more problems (not that IVAs are intrinsically bad, and they're often preferable to bankruptcy, but they have serious consequences too, and unscrupulous practitioners have been accused of bouncing people into unnecessary IVAs, when they could have negotiated a payment plan instead). Tango's reply, above, regarding the seriousness of bankruptcy (and to a lesser extent an IVA) is quite correct. DJ Clayworth's advice regarding consolidating credit card debt onto a cheaper loan is good advice too, although (especially with the current credit freeze) anyone (never mind an unemployed person) will have difficulty getting unsecured credit for such a purpose. As with insolvency practioners, beware of cowboys who try to sell you consolidation loans; again there are unscrupulous practitioners who sell you expensive loans that make your problems worse. Unfortunately there seems to be quite an industry in preying on the vulnerable overborrowed. 87.115.143.223 (talk) 21:09, 10 March 2009 (UTC)[reply]
Even if you can't get an appointment with your local CAB, and I believe many are only making appointments with priority cases in these interesting times, it is still worth contacting them. They will almost certainly have a list of recommended alternatives to themselves in the local area, appropriate to your situation. After all, you are not alone. 79.66.56.21 (talk) 23:22, 10 March 2009 (UTC)[reply]

Both are terrorist groups. Freedom of speech is usually the rationale, but couldn't you apply that to Al Queda as well? 98.221.85.188 (talk) 17:49, 10 March 2009 (UTC)[reply]

Because real politics is often inconsistant and more complicated than idealized concepts such as "rights". --Jayron32.talk.contribs 18:43, 10 March 2009 (UTC)[reply]
I'm not even sure it is illegal to join Al Quaeda, but if it is I'd imagine it has something to do with treason and enemy combatants and all that. Both these organizations though are just umbrella terms that cover many smaller groups with similar ideologies. Tomdobb (talk) 18:47, 10 March 2009 (UTC)[reply]
This actually is a very good question. This is all purely speculation on my part but it could have something to do with the fact that Al Qaeda is a foreign organization that is for all practical intents and purposes at war with the United States. Joining Al Qaeda, therefore, could be considered an act of treason. The KKK is a domestic organization and isn't really at war with the US. Another difference is that there is at least some political support for the KKK in the US and almost none for Al Qaeda. The KKK has a long history and at one time, had millions of supporters. There's also the issue of jury nullification. In the past, you had situations where all-white juries would not convict KKK members who murdered blacks. So, if you have trouble convicting someone of murder, you're probably not going to have much luck convicting someone for being a member of the KKK. Another difference is that there is no sense of urgency to outlaw the Klan. They've been around for over a hundred years and their power and influence has drastically waned. Al Qaeda on the other hand became the nation's number one threat on 9/11/2001 and there was/is a lot of fear that they will strike again in a similar fashion. A Quest For Knowledge (talk) 19:07, 10 March 2009 (UTC)[reply]
Have any cases involving the KKK been prosecuted using the anti-terrorism laws? e.g. has anyone been prosecuted for providing "material support" to the KKK? SDY (talk) 19:50, 10 March 2009 (UTC)[reply]
Yes. [1] A Quest For Knowledge (talk) 20:08, 10 March 2009 (UTC)[reply]
In my part of the world (North Carolina), people are routinely charged with "possession of weapons of mass destruction" for having things like sawed-off shotguns, so such a charge would not necessarily be particularly meaningful. --Sean 00:44, 11 March 2009 (UTC)[reply]
According to our article on him, Adam_Yahiye_Gadahn has been charged with treason, the first American to be charged with treason since 1952. A Quest For Knowledge (talk) 19:21, 10 March 2009 (UTC)[reply]
In 2003, six Americans were convicted for providing material support to al-Qaeda. Granted, I didn't read all 157 pages, but according to the indictment[2], it doesn't look like being a member of Al Qaida was one of the charges. Instead they were charged with "Conspiracy to Kill United States Nationals", "Conspiracy to Murder, Kidnap and Maim at Places Outside the United States", "Conspiracy to Destroy Buildings and Property of the United States", etc. A Quest For Knowledge (talk) 19:42, 10 March 2009 (UTC)[reply]


What about the Nazi party. Isn't it legal to join that too? We were once at war with them as well. 98.221.85.188 (talk) 19:40, 10 March 2009 (UTC)[reply]

The WW2 era Nazi party no longer exists. Although various White supremacists identify themselves as nazis or neo-nazis.Tomdobb (talk) 19:47, 10 March 2009 (UTC)[reply]
I know in parts of the European continent using the term Nazi in an organisation is illegal, but to my knowledge, in the UK it is your actions taken: possibly Incitement to racial hatred or similar. - Jarry1250 (t, c) 19:56, 10 March 2009 (UTC)[reply]
In the United States, the Nazi party isn't illegal (although it is illegal in some European countries). I don't think treason applies given that a) the United States is no longer at war with Nazi Germany and b) Nazi Germany no longer exists. In the 1930s, there used to be an official American Nazi party called The German American Bund but it effectively collapsed after American entry into WWII on the side of the Allies. According to our article, most were arrested and placed in internment camps. Also, see German_American_internment. A Quest For Knowledge (talk) 20:02, 10 March 2009 (UTC)[reply]
Under International Law, all member states are obliged to follow United Nations resolutions against designated terrorist organisations. The UN have identified the Taleban and Al-Qaeda. Thus, membership of these organisations is illegal. See [3]. (A local site, but I haven't time to google for the UN or US one). Gwinva (talk) 22:23, 10 March 2009 (UTC)[reply]
The cause and effect do not follow quite so simply. Under the UN Charter, all member states are obliged to implement (some) resolutions. In those states that have implemented the resolution, membership of the organisation may, per se, be illegal.
My understanding is that none of the SC resolutions require states to criminalise membership of the organisation per se, only to stop the funding and activities of proscribed organisations - but I'm not fully certain on this point. See also United Nations Security Council Resolution 1373. --PalaceGuard008 (Talk) 23:09, 10 March 2009 (UTC)[reply]
Apropos Nazis: AFAIK, both the KKK and AlQaida would be illegal in Germany, but not as a consequence of curtailment of "free speech" (often misunderstood as such by the uninformed), but because organizations such as these have an agenda to overturn the constitution by unconstitutional means, or have an agenda to/that compromise(s) the basic/inalienable rights anchored in the constitution.
The term for such illegal organizations is Verfassungswidrig meaning circa "anti-constitutional", and there is a branch of government -- the Verfassungsschutz, "Constitutional protection (agency)" at both federal and state level -- whose sole job it is to ensure that such organizations do not get a foot through the door. Ideally, anyway. -- Fullstop (talk) 00:36, 11 March 2009 (UTC)[reply]
I do not think joining Al Queda is illegal unless joining includes making a statement that you will plan or advocates others to overthrow the government. See sedition. The KKK, however, does not oppose the government. --Ephilei (talk) 19:37, 16 March 2009 (UTC)[reply]

Several generations of the Lords Fairfax of Cameron were born in the United States/American colonies. Albert Fairfax, 12th Lord Fairfax of Cameron, born in Maryland, was apparently finally interested enough in the peerage to move back to the United Kingdom to take over his role as a peer. Albert apparently moved back to the UK in 1917. Am I correct, then, in believing that Thomas Fairfax, 13th Lord Fairfax of Cameron was the first Lord Fairfax of Cameron to have been born in the UK in several generations? Who is the heir of Nicholas Fairfax, 14th Lord Fairfax of Cameron? Who then was a gentleman? (talk) 22:40, 10 March 2009 (UTC)[reply]

His son, Edward Nicholas Thomas Fairfax, I would think. - Nunh-huh 22:43, 10 March 2009 (UTC)[reply]
Aha, yes: [4]. Where did you find that information? Who then was a gentleman? (talk) 22:46, 10 March 2009 (UTC)[reply]
Well, I looked it up in my database :). But it's sourced to Burke's Peerage, and to the Complete Peerage, vol XIV (addendum). If you need the page numbers, I'll gladly provide. -Nunh-huh 22:51, 10 March 2009 (UTC)[reply]
Yes, please, I'm trying to add some information to the Fairfax articles and I need a reliable source. Who then was a gentleman? (talk) 22:57, 10 March 2009 (UTC)[reply]
Excellent, an excuse to use Ottobib!
Cokayne, George (1982). The Complete Peerage of England, Scotland, Ireland, Great Britain, and the United Kingdom, Extant, Extinct, or Dormant. Gloucester England: A. Sutton. p. 314. ISBN 0750901543., Mosley, Charles (1999). Burke's Peerage and Baronetage. New York: Routledge. p. 1019. ISBN 1579580831. - Nunh-huh 23:03, 10 March 2009 (UTC)[reply]
And apparently the current Lord Fairfax is no longer a Scottish Representative Peer? Who then was a gentleman? (talk) 22:45, 10 March 2009 (UTC)[reply]
All Scottish peers were admitted to the HoL in 1963, if memory serves, making election of representatives redundant. —Tamfang (talk) 21:29, 11 March 2009 (UTC)[reply]
Indeed, whatever happened to democracy eh? ;) --Cameron* 21:32, 11 March 2009 (UTC)[reply]
But I don't see his name listed on the link that I provided., Who then was a gentleman? (talk) 23:20, 11 March 2009 (UTC)[reply]

The Panizza musical dynasty

I'm looking for some information about these people. They seem to have been active in Italy in the late 19th– early 20th century. I know of at least 5 conductors with the surname Panizza: Achille, Alfredo, Arturo, Augusto and Giacomo. Depending on which source you believe, Achille, Alfredo, Arturo, and Giacomo all conducted the world premiere of Giacomo Puccini's opera Le Villi in 1884. (The details and cites are all at Talk:Le Villi#The conductor of the Milan premiere, so I won't repeat them here.)

This cite refers to Achille being a member of a "musical dynasty", which may account for the confusion about who the actual conductor of the premiere was. But my searches for information about this "dynasty", in an effort to sort out who was who and what the family connections were, have been fruitless. Can anyone help me out? -- JackofOz (talk) 22:47, 10 March 2009 (UTC)[reply]

Charles Osborne, in The Complete Operas of Puccini, says it was Achille, but he also refers to the opera as Le willis. Who then was a gentleman? (talk) 23:03, 10 March 2009 (UTC)[reply]
Does the New Grove Dictionary of Opera help at all? Who then was a gentleman? (talk) 23:05, 10 March 2009 (UTC)[reply]
This [5] also calls it Le Willis (1st version only), and has Arturo P. as conductor of versions 1 and 2. As does the Italian WP (ie Arturo P. / 1st version), but they have a red link for Arturo P. and list Giovanni Bolzoni as conductor of the second version. --Cookatoo.ergo.ZooM (talk) 23:20, 10 March 2009 (UTC)[reply]
  • Apparently its first performance on 31 May 1884 was as "Le Willis". A bit of an odd title, really. It was based on a French story called "Les Willis", which was not based on the life of the Major League pitcher Les Willis, btw, but about Slavic fairies or nymphs that go by various names such as Vila, Wila, Wili, or Veela. In French, it's Willi. Puccini's title kept the French plural "Willis", but used the Italian plural article "Le". For the revised version which premiered in December 1884, he went for the full Italian "Le Villi", and that's what it's been known as ever since.
  • Then who was a gentleman: Osborne says the conductor was Achille Panizza. OK, but how can we trust him over any of the other sources that say something different? According to the Le Villi talk page, Osborne did go so far as to deny it was Ettore Panizza, who was only 9 years old at the time. That's good, but anyone could have come to the same conclusion. What we're left with is simply an assertion by Osborne that it was Achille. What I really want is something that, rather than simply asserting that it was this Panizza or that Panizza, proves exactly which Panizza it was, and why.
  • Does the New Grove Dictionary of Opera help? I can't say, as I don't have access to a copy.
  • CookeZ: There seems very little doubt that the conductor of the revised version (26 December 1884) was none of the Panizzi but Giovanni Bolzoni, so I'd discount that cite. I'm really only interested in the premiere premiere (so to speak), and to get some more biographical material on the "Panizza dynasty". -- JackofOz (talk) 01:07, 11 March 2009 (UTC)[reply]


March 11

Please, make me stop believing in the things white supremacists say

As the supposed oracle of human knowledge, I'd like to be converted by you.--YouGiveMeTheReason...YouGiveMeControl (talk) 01:35, 11 March 2009 (UTC)[reply]

This is a reference desk, not a counselling service or psychiatric help centre. If you are experiencing delusions, you should seek help from a trained psychiatrist. If you have specific factual questions, you should ask them here. Algebraist 01:39, 11 March 2009 (UTC)[reply]
If you desire being pointed to some nice succinct works that refute many common racist claims, I recommend Gould's Mismeasure of Man as a nice start to thinking about these sorts of issues. I am sure others could recommend more. --98.217.14.211 (talk) 02:15, 11 March 2009 (UTC)[reply]
Compare Barack Obama to Larry the Cable Guy and tell me which one is the genetically inferior mud person. -- Mwalcoff (talk) 02:46, 11 March 2009 (UTC)[reply]
An intelligent advocate for that position may only argue that whites, on average, are smarter than blacks. Thus comparing any one person from each race would not be an actual objection.--droptone (talk) 12:45, 11 March 2009 (UTC)[reply]
If such a person existed then logically they would have to admit that an individual black should not be discriminated against, since they might be smarter than a given white person. That would rather ruin their supremacist argument. DJ Clayworth (talk) 13:51, 11 March 2009 (UTC)[reply]
I think the standard rebuttal for that would be that you don't make informed decisions based on the exceptions to rules. Racist logic is generally difficult to argue with, because anyone who's adopted that logic is generally working with a deeply ingrained set of assumptions, suppositions, and values which is greater than the sum of its parts. I think the best way to refute any kind of supremacist is not to pick apart their ideas, but to take a look at the various studies and experiments which show that people will buy into the weirdest crap under the sun if it allows them a feeling of superiority and control. Jane Elliott's blue eyes/brown eyes experiment is the first to spring to mind; it was summarized quite nicely in a recent Frontline video. I'm sure there's a canned white supremacist argument to shoot down said video; it might be as simple as "Nothing made by PBS is going to be unbiased." --Fullobeans (talk) 14:36, 11 March 2009 (UTC)[reply]
Also, I think Mwalcoff's comment was meant to be in jest. While Larry the Cable Guy may be a genetically inferior mud person, I don't think the intent was that either person should be taken as representative of their entire race. Tomdobb (talk) 14:40, 11 March 2009 (UTC)[reply]
If you're serious about wanting to be converted, you already know that what they say is hateful bullshit, and you're just looking for an excuse to make that leap away from a bunch of scared troglodytes. So go for it. -- Captain Disdain (talk) 09:20, 11 March 2009 (UTC)[reply]

Note: this is not a troll. I (not the original poster) really have the problem I describe in this comment I am about to describe: the same problem you do, but not about my race's "supremacy" but the awful habits Jews have, ie my impression of the same. I only acquired this disdain by pretending to be a Jew, infiltrating their synagogues and social networks in several cities, and coming to have a problem with how the Jews' network operates throughout the world. My problem is much bigger than yours, because my problem was in my mind based on facts, regularly reported in the New York Times, in stories such as today's "Madoff Will Plead Guilty; Faces Life for Vast Swindle". So how did I stop having my problem, in my mind based on facts? Well, in my case the solution is to focus on how hot jews are, despite the awful things I thought they did. You could try doing the same thing -- find hot representatives of the races you find odious, probably at universities etc, and by spending time with them you will come to love their race despite your understanding of their race's habits. Unlike me, you might also benefit from more exposure to the race, in case your understanding of the facts would change and you would realize how wrong you were just by spending time with them - in this case you could pretend to be one of them and infiltrate their race, and thereby you can try coming to a more correct factual understanding. (However, as it happened in my case, you might only make the problem worse, if mere exposure to the real habits of the race does not change your opinion of them for hte better). Therefore I recommend the route I describe above -- instead of trying to address your rational thinking, by gathering more facts through living with them first-hand, you could speak to your own sense of beauty and style, by finding outstanding representatives and focusing on them. Good luck. We do NOT need hate in the world, and I hope you are able to overcome your hate as I was able to overcome mine. note: first this comment was reverted as hate speech - I do NOT hate the Jews -- I USED to have a problem with this, but was able to overcome it, and I would like to help YOU overcome yours, which is what this whole question is about. Again, the problem was with me, not the jews, and the problem is with you, not whoever you consider the inferior races. Then this comment was reverted as a troll, but it would be a troll if I were only just pretending to have had this problem and bene able to overcome it: in fact, this is just what happened. I don't see how I can act in any better faith or be more informative than I have been. Of all the responses you have received mine is the only first-hand account of a successful solution to the problem I used to have, which is the same one you have now. Thank you and God Bless. 92.230.65.185 (talk) 14:54, 11 March 2009 (UTC)[reply]

If you are not a troll, this is indeed not a counseling center. However, if you are thinking thoughts despite your best efforts not to think them, that is a fairly classic definition of obsessive thinking. You should get medical help. If this is not what you are describing, I admit I have difficulty understanding your point. Which may be another indicator of mental health issues. --Moni3 (talk) 15:00, 11 March 2009 (UTC)[reply]
Your comment could be in response to the original question - "Please, make me stop believing in the things white supremacists say". The one thing my comment does that yours doesn't is to actually give the poster tools for stopping believing in white supremacy. I told him: he needs to spend time with hot, stylish, outstanding representatives of the "inferior" races. What do you do? Send him to get medical help. As for whose response is more useful, the original poster can decide. 92.230.65.185 (talk) 15:12, 11 March 2009 (UTC)[reply]
Quite. I thought the issue was yours. Difficult to understand indeed. Hot, stylish, outstanding representatives of inferior races? I missed that. How unfortunate. --Moni3 (talk) 15:42, 11 March 2009 (UTC)[reply]
sorry, I can see how the comment is unclear. I changed it to clarify that I'm not the original poster but someone who has found a solution to this type of problem.
Think of people as individuals rather than members of some arbitrary group. If you're white, there are millions of things that white people have done wrong - Holocaust, slavery, Spanish Inquisition, slaughter of Native Americans, etc.. None of that is your fault because you are an individual. A Quest For Knowledge (talk) 16:07, 11 March 2009 (UTC)[reply]
In fairness, Larry the Cable Guy is actually a comedian named Daniel Lawrence Whitney who is simply playing a make-believe character. :) Pastor Theo (talk) 03:12, 13 March 2009 (UTC)[reply]
Yeah better to compare the previous president and the cur..... ooops did I say something? Nil Einne (talk) 21:51, 17 March 2009 (UTC)[reply]
A Class Divided is, PBS claims, one of its most requested documentaries, and follows an Iowa teacher as she gets her all-white class to experience discrimination and prejudice, and then debriefs them. Full details here. Jane Elliott has continued this, by training adults in what racism feels like, since 1968. She would appear to have some success in helping people break their mental chains. BrainyBabe (talk) 09:30, 14 March 2009 (UTC)[reply]

Blood donation in the third world

I've searched high and low for good references on this, and I can't find anything that satisfies me. I know from various sources that in more or less the entire wealthy world (Western or not), the standard is to have blood donations from folks who volunteer to donate for a community supply. I've read some references, albeit not very helpful ones, that make comments about "replacement donations" being common when that kind of established volunteer supply is unavailable. Most are written with the assumption that the reader knows what they're talking about, and I really only understand the process for the US.

This, as far as I know, could mean one of two things. Let's say Uncle Phil needs a transfusion. You could either 1) donate a unit of blood that is transfused to Uncle Phil or 2) a unit of blood is taken from the supply and you give a unit to replace it and the supply remains at the same total number of units. The first seems more, the second is more likely given the vicissitudes of transfusion medicine where setting up two closely-related people can have grave implications even if the types match (typically not an insurmountable barrier, but one that is sometimes difficult). Which is it? Is it both? What kind of reliable sources are available on the practice? SDY (talk) 02:23, 11 March 2009 (UTC)[reply]

First, I'm not sure what the "third world" in the title refers to: you seem to be discussing the wealthy (western) world. Secondly, the logistics of blood screening and general processing mean that Uncle Phil will get stock blood, and your blood will be added to the general pool. In the unlikely scenario that you are the only match for Uncle Phil, he will have to wait a little while for his transfusion. Even then, your blood will not be marked "For Uncle Phil". It will be typed and processed, and he might nor even get it. Gwinva (talk) 02:32, 11 March 2009 (UTC)[reply]
The "replacement" thing is essentially about billing. The blood recipient (or his insurer) is charged for both the blood he receives (often billed as a "replacement fee") and the various processes entailed in receiving it; if he or others in his name "replace" the blood he is charged only for the processes. Donating blood for elective procedures and earmarking it for a specific recipient is another unrelated issue. - Nunh-huh 02:41, 11 March 2009 (UTC) (That is, using your example: Uncle Phil has already had a blood transfusion. He can either pay the replacement fees, or have someone donate the units in his name.) - Nunh-huh 02:43, 11 March 2009 (UTC) (Here's a ref, but it's mind-numbingly dull: [6]) - Nunh-huh 02:48, 11 March 2009 (UTC)[reply]
That's for the US, which isn't relevant to my question. I'm looking for what happens in countries covered by PEPFAR, for example. The two examples of processes used that I gave exist in the developed world that might be used in place of an established supply, but I do not know if they are analogous to those used in developing countries or which "replacement donation" might apply to. SDY (talk) 03:01, 11 March 2009 (UTC)[reply]
So you wonder, then, whether "replacement donation" for some reason means something different in the 3rd world than it does in the 1st? - Nunh-huh 04:33, 11 March 2009 (UTC)[reply]
How the logistics work, and how much of it is "replacement" as opposed to "directed" is unclear in the sources I've seen. I'm not looking for definitions, I'm looking for practices. SDY (talk) 15:33, 11 March 2009 (UTC)[reply]
It is your second alternative. A unit of blood is taken from the supply, and the patient is requested to ask a friend or relative to donate blood and thereby replace it. See here. --NorwegianBlue talk 20:54, 11 March 2009 (UTC)[reply]
Excellent, thanks. "Donations from family" in the US almost always means directed donations (the first option) and this is why I was leery of polevaulting to conclusions on this. SDY (talk) 21:11, 11 March 2009 (UTC)[reply]

Refugees into Canada

Does anybody know how many refugees from Africa and South Asia, are admitted into Canada each year? Sonic99 (talk) 02:38, 11 March 2009 (UTC)[reply]

I've been looking around, but breakdowns of refugee origin by geographic area are hard to track down. I found this document which gives some total numbers (147,000 total refugees settled in Canada from 2002 to 2007, or an average of 29,400 per year) and the top ten countries of origin: in order, Afghanistan, Colombia, Ethiopia, Burma, Sudan, Democratic Republic of the Congo, Somalia, Iran, Eritrea, and Iraq. I'll keep digging. - EronTalk 03:48, 11 March 2009 (UTC)[reply]

Statistics Canada addresses this subject to some extent in the annual Report on the Demographic Situation in Canada. The latest issue covers 2005–2006 (mostly, with some data from later years) and is available in PDF here or in sections in HTML under the other link. Unfortunately, while it provides the total number of refugees each year, its table of immigrants by category and country (table 4.3) only covers the 10 countries providing the most total immigrants, not the most refugees. Here's what that table shows for 2007, by the way:

Country of birth Economic Family Refugees Other Total
China and Hong Kong 16,338 10,367 1,583 608 28,896
India 15,335 11,988 848 349 28,520
Philippines 15,191 4,135 36 356 19,718
Pakistan 5,342 2,763 1,324 379 9,808
United States 4,371 3,239 424 716 8,750
Great Britain 5,523 1,579 16 206 7,324
Iran 4,730 1,453 847 165 7,195
South Korea 4,820 790 45 254 5,909
Colombia 1,177 541 3,544 120 5,382
Sri Lanka 747 1,499 1,088 734 4,068

But as you see, that accounts for less than 10,000 refugees or about 1/3 of a typical year's total.

The same report for 2003–2004 comes irritatingly close to answering the question: at one point it says that "four African countries appear on the list of the 10 countries that account for the most refugees in Canada, namely Sudan, Zimbabwe, Congo and Somalia. Close to 5,000 refugees came from those countries in 2004, representing approximately 15% of the 32,700 total." It also talks specifically about refugees from the Balkan countries. But no actual list is given.

--Anonymous, 04:45 UTC, March 11, 2009.

Possible scenarios of consequences for an immediate withdrawal of ISAF from Afghanistan

Assuming that all foreign troops in Afghanistan left immediately what would the possible outlook for Afghanistan be? A list of most likely scenarios would be nice. If you know of any good links where experts discuss likely outcomes that would also be appreciated. ExitRight (talk) 03:23, 11 March 2009 (UTC)[reply]

Speculation is not what the reference desk is about. But perm any from anarchy, tribalism, taliban, civil war. And you cannot consider it in a vacuum; it probably depends a lot on what Pakistan, Russia, Iran and others do next. --Tagishsimon (talk) 07:13, 11 March 2009 (UTC)[reply]
"perm" ? StuRat (talk) 14:47, 11 March 2009 (UTC)[reply]
The Kabul government of Hamid Karzai wouldn't last for long. Afghanistan has traditionally been battled over by their neighbors. Russia likely still has a sour taste from the last time they tried to intervene there, so would probably stay out. That leaves Pakistan and Iran to fight over the Afghan's fate. Since Pakistan is almost a failed state and near civil war itself, I'd expect that Iran, being much stronger since Iraq is no longer a threat, would gain the upper hand eventually. Although, when Iran does start to gain control, that might provide a new incentive for those in Pakistan to work together to defeat them, much as they did to defeat Russia there. We could even conceivably see a time when the US would again support the Taliban, if Iran is taken to be the more serious threat. StuRat (talk) 14:47, 11 March 2009 (UTC)[reply]
"perm, v2 colloq. (orig. Brit. gambling.) To make a selection of (a number of items) from a larger number; to make a permutation of, to subject to permutation." (OED). See also Football pools#Winning. --ColinFine (talk) 00:15, 13 March 2009 (UTC)[reply]
Thanks. That's the first time I've heard it abbreviated and used as a verb like that. StuRat (talk) 07:01, 14 March 2009 (UTC)[reply]

Arguments for blind faith

Has anyone attempted to prove through logical argument that blind faith is necessary or desirable? NeonMerlin 05:21, 11 March 2009 (UTC)[reply]

Pascal's Wager proposes that one should decide to believe in God because if you're right, you win, and if you're wrong, it doesn't matter. - EronTalk 05:28, 11 March 2009 (UTC)[reply]
You only win if your specific religion would be right, imagine the Catholic finding out and jesus was actually a practical jokster who with the help of his 12 henchmen preformed magic shows in the middle-east, and that Judaism was right, they wouldn't win, would they? ;-) chandler · 15:05, 11 March 2009 (UTC)[reply]
Well, I didn't say it was a particularly convincing logical argument... - EronTalk 16:16, 11 March 2009 (UTC)[reply]
The table at Absurdism#Relationship_with_existentialism_and_nihilism lists some different approaches to this question and points to articles about them. 207.241.239.70 (talk) 07:27, 11 March 2009 (UTC)[reply]
The clearest argument is Brain in a vat and related arguments. Essentially we have no guarantees that anything we perceive is actually the case; and since all reasoning depends on the human mind (which is fallible) we actually have no guarantees that anything we deduce is correct. Most people adopt the 'blind faith' position that a) there is a reality and b) what we perceive more-or-less corresponds to it. DJ Clayworth (talk) 13:48, 11 March 2009 (UTC)[reply]

A non-narrative novel that wasn't mean to be read from start to finish?

At some point, I heard about a printed book, which I'll loosely call a novel, which had a structure which encouraged the reader to "dive in", midway through the work, and piece together for themselves the story.

To further probe my hazy memory-- I believe (or else I previously speculated) that the work was somehow analogous to a document dump of some kind-- a series of notes, memos, or files that were presented out of chronological order. Parts of the book would make reference to other parts of the book-- a reader would have to decide for themselves what to read and in what order. When another sub-section of the book was referenced, a reader might choose to go look at the reference sub-section; or they might choose to ignore the reference and continue on reading their initial section; or they might jump back and forth, or might even just randomly turn to any page in the book and try to start parsing the text.

In this way, every reader of the book had a different experience, since ever reader chose their own path through the work.

Does this ring any bells? The concept is really fascinating and I'd love to read the book in question. --Alecmconroy (talk) 09:41, 11 March 2009 (UTC)[reply]

Might it be Milorad Pavic's "lexicon novel" Dictionary of the Khazars? Malcolm XIV (talk) 09:52, 11 March 2009 (UTC)[reply]
(ec) Could this be Choose your own adventure books? Although, these books would always start at the beginning - you couldn't just open anywhere and start reading, but they would eventually say something like "Does Jimmy run away? If yes, turn to page 47, If no turn to page 16" Rfwoolf (talk) 09:54, 11 March 2009 (UTC)[reply]
It sounds like an epistolary novel. A particularly labyrinthine example of that, brought to mind by your description, would be House of Leaves by Mark Z. Danielewski, with its footnotes chasing each other and the book generally being a maze of sorts, although you probably couldn't just jump in anywhere. -- Captain Disdain (talk) 10:34, 11 March 2009 (UTC)[reply]
Julio Cortázar's Rayuela is another example of this sort of metafictional work. There are a number of them; I believe I've even seen a mystery "novel" consisting of a box of facsimile documents and other clues, from which one is to reconstruct the mystery and solution. Deor (talk) 13:46, 11 March 2009 (UTC)[reply]
Probably one of the crime dossiers of Dennis Wheatley.--Rallette (talk) 19:17, 11 March 2009 (UTC)[reply]
Could be something by BS Johnson, such as House Mother Normal or The Unfortunates. --Richardrj talk email 13:55, 11 March 2009 (UTC)[reply]
While this may not be what you're thinking of, Lemony Snicket: The Unauthorized Autobiography is another example of that sort of book(a collection of letters, photos, news clippings and the like, often with notes scrawled on them by the compiler). 69.224.37.48 (talk) 18:57, 14 March 2009 (UTC)[reply]

Erotic fiction gamebook

The above question made me think of another. I remember reading an interesting example of the genre of gamebooks, which was I think translated from the French and was possibly written by a woman. It was an erotic kind of adventure, but very literary in style. Does anyone know what the book might be? List of gamebooks is no help, by the way. --Richardrj talk email 10:39, 11 March 2009 (UTC)[reply]

You might try trawling through the extensive list of gamebook series here to see if anything rings a bell. (I notice that there is a series, of two books, called Create Your Own Erotic Adventure, with one of the books being titled The Classics Professor, which has to be among the least-promising ideas for an erotic adventure I've ever heard of.) Deor (talk) 13:26, 11 March 2009 (UTC)[reply]
Maybe the prof specializes in Catullus. —Tamfang (talk) 21:13, 11 March 2009 (UTC)[reply]
I've known a couple of those—not erotic-adventure material, though. Deor (talk) 01:01, 12 March 2009 (UTC)[reply]

sex offender crastration

There's a New York Times article today, Europeans Debate Castration of Sex Offenders that lists a lot of arguments for and against castration of sex offenders. But when I read

I wondered: wouldn't a simple solution be to offer voluntary castration services, to people who have not yet committed any crime? Then that objection disappears, and all the arguments about "my body my choice" that pro-choice people use regarding abortion would apply. Why isn't this a choice that is even offered? Many sex offenders know full well that acting on their urges are illegal and unethical. Why not give them a choice at castration clinics?


92.230.65.185 (talk)

Can they not do it voluntarily? I doubt many doctors would do it without sending you for a psychiatric exam first, but I can't see why they would refuse if you are clearly of sound mind and know what you're doing. It would be similar to amputations to cure Body integrity identity disorder, which some doctors will do (although not without some controversy, admittedly), and in this case there is the added benefit of protecting others, so I would expect even more doctors to be willing to do it. --Tango (talk) 12:51, 11 March 2009 (UTC)[reply]
This is really a call for debate, which is not what the reference desk is for. DJ Clayworth (talk) 13:43, 11 March 2009 (UTC)[reply]
According to this decidedly not work-safe article in another wiki, "Castration by a doctor is, by far, the safest method of castration, and potentially one of the hardest to achieve. Most doctors will absolutely refuse to perform a voluntary castration upon a patient. However, there are a few medical professionals out there who are open to such ideas. The difficulty lies in both locating them and obtaining the funds for travel as well as the fees they charge. Generally, their fee ranges is $1200-2500." The article deals primarily with voluntary castration for fetish-y purposes; conceivably, it might be easier to find a doctor willing to perform the procedure on a would-be sex offender, but who knows. --Fullobeans (talk) 13:53, 11 March 2009 (UTC)[reply]
I'm not sure I buy it that castration will stop pedophiles. After all, there are women who have sex with children, and they don't have testes. StuRat (talk) 14:37, 11 March 2009 (UTC)[reply]
I didn't think so either but according to Castration: "Physical castration appears to be highly effective as, historically, it results in a 20-year re-offense rate of less than 2.3% vs. 80% in the untreated control group, according to a large 1963 study involving a total of 1036 sex offenders by the German researcher A. Langelüddeke, among others, much lower than what was otherwise expected compared to overall sex offender recidivism rates." The equivelent of castration in females would be Oophorectomy. A Quest For Knowledge (talk) 14:58, 11 March 2009 (UTC)[reply]
The testes produce the chemical that warps men's minds. I remember an interesting interview on This American Life with a female-to-male transsexual who said that as soon as she got on testosterone (at doses much higher than what a normal (non-15-year-old) male receives naturally), a torrent of pornographic fantasies flowed through her mind (not unpleasantly) during all waking hours. --Sean 16:14, 11 March 2009 (UTC)[reply]
Sexual drive is not the first motivator of rape and other forms of sexual abuse: power is. Castration is simply a medieval punishment, like blinding. Blinding might be equally effective, don't you think? You might cut their tongues out at the same time, no?--Wetman (talk) 20:12, 11 March 2009 (UTC)[reply]
Do you have studies to back up its effectiveness? (Or your claim of "first motivator? citation needed) Rmhermen (talk) 20:36, 11 March 2009 (UTC)[reply]
If you want citations, you may want to read our article on motivation for rape, for starters. Wetman is correct in that most of the time rape is all about the power dynamic... about humiliation and rage. Especially in cases of the rapist lurking in the bushes and assaulting and raping a passersby, or doing the same to a co-worker or some other acquaintance, that's really not about getting laid. (Statistically speaking, very few rapes are committed by complete strangers.) Likewise, rape that takes place within a relationship or by someone you know well is certainly more about subjugating and humiliating someone or the rapist needing to feel in control than about the actual sexual gratification. That said, simply wanting to have sex can absolutely be the motivator in certain situations, especially in (all-too common) cases where the rapist's judgment is impaired by alcohol and they ignore verbal protests, but don't really resort to violence as such. There's a difference between having appallingly bad judgment and actual malevolence. And I'm not in any way excusing the former, I should probably stress. I just don't think that it's psychologically the same thing as the latter. -- Captain Disdain (talk) 22:32, 11 March 2009 (UTC)[reply]
But oftentimes sex is about power, too. So if rape is about power, it is at least as much as sex is about power, so rape is at least correlative with, if not absolutely about, sex. In other words, I don't buy the "rape isn't about sex" part of the argument. --PalaceGuard008 (Talk) 00:12, 12 March 2009 (UTC)[reply]
I think you misunderstand. Of course rape has a strong sexual element to it, but it's not about sexual gratification (at least in the sense that someone desires an orgasm and resorts to rape to get it). Generally speaking, people don't commit rape out of a desire for sex. Sexual urges tend to shape the urges they feel, sure, but they don't rape someone because they want to have sex. Or to be more blunt about it: "boy, I can't get laid, I'm just gonna have to rape someone 'cause I really need to have sex" may sometimes be the driving force behind a rape, but in the majority of cases, that's nowhere near what it is about. Typically, the rapist is likely to find the rape arousing (and in some cases may even require it to climax), but he's not turned on by the sex itself, he's turned on by the power dynamic, the authority, the victim's pain and fear, by the power fantasy, etc.
You're correct in that sex is oftentimes about power, too, though. That's certainly the case with BDSM, and it's often an element in very conventional sexual relationships, too. But there's a world of difference between acting out a consensual fantasy and actually forcing someone into sex against their will; the power relationships in consensual sexual encounters are not similar to the power relationships in rapes, for the simple reason that there's an element of trust and intimacy in the former, and a violation of trust and unwanted intimacy in the latter. -- Captain Disdain (talk) 01:28, 12 March 2009 (UTC)[reply]
Isn't it possible that someone "forces" someone else to have sex with them because of a perceived or real inability to "persuade" someone else to have sex with them? Bus stop (talk) 01:44, 12 March 2009 (UTC)[reply]
That's still about power, Bus stop. What is "force" but a demonstration of power? It is someone saying my right to have sex with you is more powerful, more important, more justifiable than your right not to have sex with me, to be unmolested and safe. Mixing up sex and violence is pretty creepy without complete agreement from all parties. And there is no "right" to have sex, not for anyone, in any circumstance, ever. // BL \\ (talk) 01:51, 12 March 2009 (UTC)[reply]
When a bank is robbed, some sort of power has to be wielded by the bank robber. But the act isn't necessarily performed in order to express one's powerfulness. Bus stop (talk) 14:50, 12 March 2009 (UTC)[reply]
Absolutely. But that's a lot more about the perpetrator's feelings of inadequacy -- that is to say, the power dynamic -- than sexual desire. I mean, if what you really, really want is just sex, there are always prostitutes, and in any case, the urge to, uh, get some just isn't overwhelming enough for the vast majority of men to completely lose all control and sense of right and wrong when they get in the mood.
I'm not saying that no one has ever committed a rape just out of a plain desire to have sex with someone, of course. Sure that's happened -- I guess the most typical example would be a situation where the rapist takes advantage of a victim who's passed out, or the rapist doesn't really fully understand the consequences and implications of his actions because of temporarily impaired judgment or downright stupidity, or -- in some cases -- an inability (real or intentional) to correctly interpret the reactions of a very passive, but unwilling victim. "Well, she said no a couple of times and now she's just lying there rigidly but not resisting or responding, so I guess I can get away with doing this 'cause I really want to have an orgasm." But that's really not what rape is about most of the time. -- Captain Disdain (talk) 02:05, 12 March 2009 (UTC)[reply]
Yes it's "about" power. That doesn't mean it's not "about" sex. It's a false dichotomy to say that this is about power, therefore it is not about sex. Rape isn't really "about" anything. It's sex, it's power, and a whole host of other issues. --PalaceGuard008 (Talk) 05:19, 12 March 2009 (UTC)[reply]
Right. But we're talking about what primarily motivates your average rapist to actually, say, beat someone up and have sex with them against their well, and it's not the desire for sex -- it's power, aggression, all that. They are aroused by that, sure, but the sexual element is more the effect than the cause. And yes, that's a generalization, but it's a fairly accurate generalization. I mean, I'm not just saying that or theorizing here, there's plenty of research that backs this up. (It's not uncommon for men to not even ejaculate during rape.)
The statistics also support this: according to our article on rape, only 2% of all rapes are committed by strangers, whereas over 50% of them are committed by people who are not just acquaintances, but actually close friends, spouses or the like. In a relationship like that, where the rapist knows the victim well, surely it's not primarily about getting their rocks off -- if it was just about the urge to get laid, they'd find someone else instead of hurting someone they care about. The fact that almost all rapists choose people they know is very telling.
(I should probably add that I know there's the classic hardline feminist view of rape, which is, basically, that all sex is rape and it's always about the power male-centered society wields over men, and that's not what I'm talking about. That's a political view, not a psychological one.)
Anyway, I don't really want to get into a much deeper argument than this about the subject -- the research and psychology strike me as conclusive enough to make that kinda pointless. We can agree to disagree. -- Captain Disdain (talk) 10:40, 12 March 2009 (UTC)[reply]
Captain Disdain, while you have been very loquacious (but cited no sources), you have completed failed to address the topic of why at least one scientific study has shown castration to be effective. Rmhermen (talk) 15:51, 12 March 2009 (UTC)[reply]
Killing sex offenders would be 100% effective at stopping them from committing the crime again, and yet we don't do that. There is a certain measure of hmanuity even towards the convicted in most civilized societies; regardless of whether or not castration could be viewed as effective is moot when one considers whether or not it would be just or right for us to do it. --Jayron32.talk.contribs 17:07, 12 March 2009 (UTC)[reply]
There's chemical castration. That might be more humane. BTW, if certain offenses can be considered the result of a mental illness (such as pedophilia) can the state order someone to take their medication? A Quest For Knowledge (talk) 17:53, 12 March 2009 (UTC)[reply]
Our article on motivation for rape has sources, as I already said. As for castration's effectiveness, it's true I haven't addressed that. I admit I wasn't even trying to. Sorry, my bad! In any case, I'm sure castration can be effective much of the time in curtailing pedophilia (and perhaps even rape), but that may be a short-sighted and kind of narrowly defined success. Having a sexual preference for children is unfortunate, and my heart goes out to those poor bastards who, through no fault of their own, suffer from it, because that's lousy hand to be dealt -- if that's how you're wired, you're up for what is likely to be a miserable existence. But my sympathy evaporates pretty quickly when someone acts on that urge.
Now, if you're a person of moral fiber with a tendency towards pedophilic behavior, eliminating your sexual urges altogether strikes me as an understandable solution. I don't know if it's a good solution, but if you just want to get rid of that urge, volunteering to be castrated kind of makes sense. But you may be getting out of the frying pan and into the fire; the psychological problems brought on by such a procedure may outweigh the benefits, at least for the individual in question, but possibly also for the society. In any case, I don't think pedophiles who are that active in seeking solutions for their problem are at the heart of the pedophile problem. Even then, you're not really dealing with the cause, because that's really between the ears, not the legs.
As for forced castration, that has implications other than just whether it works. I mean, you can keep a thief from stealing by chopping off his hands, but that doesn't mean it's a good solution. It's not just a question of coming up with a procedure to stop someone from doing something ever again -- that's not difficult. The challenge lies in coming up with a solution that is also humane and constructive -- in this case, one that keeps people who already tend to feel alienated both by society and even their own urges from becoming even more so. To exaggerate a little, you could put it this way: if you mutilate someone badly enough, that'll probably either break their spirit or drive them to excessive behavior. -- Captain Disdain (talk) 19:09, 12 March 2009 (UTC)[reply]

Have any two warring nations ever managed to simultaneously capture each other's capital?

Have any two warring nations ever managed to simultaneously capture each other's capital? The closest I can think of is the Korean War. North Korea had captured Seoul, South Korea's capital in June 1950. In Sept, MacArthur launched a successful amphibious invasion. Had MacArthur decided to drive north instead of south, he might have been in a position to occupy North Korea's capital while North Korea occupied South Korea's capital. Obviously, it didn't happen that way but I'm wondering if there's ever been a situation where two warring nations managed to simultaneously occupy each other's capital. A Quest For Knowledge (talk) 15:32, 11 March 2009 (UTC)[reply]

you mean capital, right? —Preceding unsigned comment added by 92.230.65.185 (talk) 16:00, 11 March 2009 (UTC)[reply]
Yes, I did. Thanks for the correction. I fixed my OP. A Quest For Knowledge (talk) 16:09, 11 March 2009 (UTC)[reply]
Not exactly the situation you asked for, but when at the end of WW2, Berlin surrendered to the Allied forces (Russians mainly) on May 2nd, Germany still occupied the capital of the Netherlands (and perhaps a few other allied countries like Denmark?) for a few days. I can't think of another example immediately. Fram (talk) 15:31, 13 March 2009 (UTC)[reply]
Well, no one else has managed to come up with anything, so I'm guessing it's never happened. A Quest For Knowledge (talk) 22:25, 13 March 2009 (UTC)[reply]
The only thing close to that I can think of was during the Second Punic War. Hannibal's army had virtually conquered Italy, but hadn't quite been able to take Rome itself, meanwhile the Roman army had snuck round through Spain and turned up just outside Carthage. Though in that situation neither side did quite manage to get into the capital itself, but they did both get very close. 148.197.114.165 (talk) 17:44, 15 March 2009 (UTC)[reply]

At the end of World War II, Japan surrendered to the Allied Powers in August, 1945, and the Allied forces were in full control in Tokyo by 30 August 1945 at the latest (date of arrival of MacArthur and official start of the Occupation of Japan). While the occupation force was largely American, there was a nominal contingent from the Republic of China, including a commander of the Chinese occupying forces in Japan.

Japanese troops in China officially surrendered to China on 9 September 1945. Until then, Japan was in full control of the Chinese capital by law, Nanjing. The actual transfer of control took some further time to effect.

So, during those 9 days at least, arguably Japan and China were simultaneously occupying each other's capitals. --PalaceGuard008 (Talk) 22:31, 15 March 2009 (UTC)[reply]

What's the term for this feeling?

I'm thinking of the uncomfortable feeling you sometimes get when you read or hear an argument that wildly contradicts what you know(or at least firmly believe), or which you find highly offensive, and yet find it logically and rhetorically convincing. 69.224.37.48 (talk) 15:48, 11 March 2009 (UTC)[reply]

Cognitive dissonance. Being able to believe the two contradictory ideas without the uncomfortable feeling is doublethink. 87.115.143.223 (talk) 16:05, 11 March 2009 (UTC)[reply]
That sense of loyalty applied to an orthodoxy is a symptom of induced cult behavior, by which cults may always be recognized, irrespective of their political power and the number of their followers.--Wetman (talk) 20:07, 11 March 2009 (UTC)[reply]

Polands last queen?

Was Elżbieta Szydłowska married to king Stanisław August Poniatowski? And if so, why was she not a queen? Was it a morganatic marriage, and if so, why? --85.226.44.201 (talk) 17:47, 11 March 2009 (UTC)[reply]

Both of their articles state that they were married and that it was a morganatic marriage. The articles don't state why, but reviewing the morganatic marriage article suggests that it was because they were not of the same social rank. He was born a Count; she was the daughter of a szlachta- EronTalk 18:14, 11 March 2009 (UTC)[reply]

Toronto demographics

I know that Crescent Town neighbourhood is home to Toronto's largest Bangladeshi-Canadian community, Thorncliffe Park is home to Toronto's largest Pakistani-Canadian community, Woburn is home to Toronto's largest Indian-Canadian and Sri Lankan-Canadian community, Don Valley Village is home to Toronto's largest Iranian-Canadian community, O'Connor-Parkview is home to Toronto's largest Afghani-Canadian community, and Kingsview Village - The Westway neighbourhood is home to Toronto's largest Somalian-Canadian community. So, what about which neighbourhood is home to Toronto's largest Arab-Canadian community?, what about which neighbourhood is home to Toronto's largest Turkish-Canadian community? what about which neighbourhood is home to Toronto's largest Tajik-Canadian community? what about which neighbourhood is home to Toronto's largest Azeri-Canadian community? what about which neighbourhood is home to Toronto's largest Kazakh-Canadian community? what about which neighbourhood is home to Toronto's largest Uzbek-Canadian community? what about which neighbourhood is home to Toronto's largest Turkmen-Canadian community? what about which neighbourhood is home to Toronto's largest Kyrgyz-Canadian community? —Preceding unsigned comment added by 142.204.74.89 (talk) 18:00, 11 March 2009 (UTC)[reply]

Please don't automatically generate your questions. It's offputting, to say the least.
If you'll go here and click on a neighborhood name, then on "Social Profile #2 - Language & Ethnicity", you'll be able to see the ethnic makeup of each neighborhood. You can also click on an ethnicity here, under "Ethnic Origins for Toronto CMA", and see a map of where those people live in Toronto. --Sean 20:22, 11 March 2009 (UTC)[reply]

Which other nationals are Americans most similar to in character?

I do not mean similar in GDP or language etc., but in personality, personal style, or character. My guess is Germans. 78.147.91.173 (talk) 22:39, 11 March 2009 (UTC)[reply]

It would depend on what kind of Americans you were talking about. Folks from Pacific Northwest and the Scandinavians would probably line up well, some parts of California with Australia, others with Mexico, Texans and Argentines, Louisiana with those hexagonal folks, etc... Canadians are probably the closest culturally if you absolutely had to choose, but there are some pretty substantial differences. I've actually heard advice for Americans traveling internationally to say that they are from Canada because of the stereotypes about people from the US, which is probably bad for the Canadians. SDY (talk) 23:09, 11 March 2009 (UTC)[reply]
English-speaking Canadians are pretty much the same as Americans, much as they'd like to deny it. -- Mwalcoff (talk) 04:01, 12 March 2009 (UTC)[reply]
Definitely not Germans. Well, maybe some folks from the upper east coast might come closer. There are worlds of differences, though. 76.97.245.5 (talk) 14:55, 12 March 2009 (UTC)[reply]
This appears to me to be the worst sort of opinion gathering, looking for a confirming of stereotypes. While I am pleased that no one has explained the "why" of their opinion, I really don't think this is Ref Desk material. // BL \\ (talk) 17:10, 12 March 2009 (UTC)[reply]
I agree with Bielle that a direct, opinion-based answer to this question isn't appropriate for the Desk. There are some resources out there that measure international public opinion, though, and the OP might get some of the insight she is seeking into "national character" by reviewing what surveys show about attitudes and opinions in different countries. The Pew Global Attitudes Project is probably a good place to start. - EronTalk 17:25, 12 March 2009 (UTC)[reply]
Seriously. Germans all look like Klaus Nomi. Well ok, no. But it's hard to find similarities even between Americans sometimes; a Chicano in Fresno may have very little in common with a Swedish-American housewife in Minnesota, who will probably not identify too strongly with a Chasid from Williamsburg. Class and ethnic distinctions, even within the same city, can be striking. But if you pick a very stereotypical Joe American, I think he will, regardless of his ethnic background, have more in common with most Europeans (and those from strongly European-influenced nations) than with most of the rest of the world. He'll probably identify most readily with the residents of industrialized former British colonies with high standards of living, and could sit down at a table with residents of Johannesburg, Toronto, Melbourne, Brighton, Glasgow, and Dublin with only minor culture shock. GDP and language, I think, have a lot more to do with personality, personal style, and character than we like to admit. If I had to pick one country, though, and if we ruled out Canada as the obvious choice, I'd pick Australia. And if we ruled out all the Commonwealth Nations on principle, then it gets more difficult, but I'd pick the Netherlands over Germany. --Fullobeans (talk) 17:35, 12 March 2009 (UTC)[reply]
Is it correct that the greatest number of immigrants to the US came from Germany? Not England? 89.243.40.164 (talk) 00:31, 13 March 2009 (UTC)[reply]
Our articles on German Americans and English Americans claim 8 million Germans and only 3.5 million English immigrants to the U.S. But the yearly figures mentioned in the article seem like they should add up to a higher number for English immigrants - and, of course, it doesn't include Welsh, Scottish, etc. Rmhermen (talk) 06:17, 13 March 2009 (UTC)[reply]
The latter article actually says 3.5 million English immigrants after 1776; there were already 2 million in the country, for an estimated total of 5.5 million. English immigration was apparently steady throughout the 18th and early 19th centuries, then peaked in 1842 and subsided. So that's a good long period of fruitful multiplication prior to the German immigration boom of 1848-1914, which brought 6 of those 8 million Germans to the country. That, combined the absence of Welsh/Scots/Irish from the tally, could explain the seeming dearth of English folks from the immigration register. --Fullobeans (talk) 07:52, 13 March 2009 (UTC)[reply]
Anyone care to explain SDY's characterization of the French as being "Hexagonal"? Astronaut (talk) 00:26, 13 March 2009 (UTC)[reply]
Geographical shape of the country? --PalaceGuard008 (Talk) 00:42, 13 March 2009 (UTC)[reply]
I have a French textbook that actually draws a heavily-inked hexagon around a map of the country, because apparently the hexagonality of France is of crucial importance to students of the French language. --Fullobeans (talk) 07:52, 13 March 2009 (UTC)[reply]
'The hexagon' is a term used to refer to metropolitan France, as it's broadly the shape of... well, a hexagon [7] --Saalstin (talk) 14:04, 13 March 2009 (UTC)[reply]
Yes, and as such features on the French euro coins. BrainyBabe (talk) 15:45, 13 March 2009 (UTC)[reply]
L'hexagone is a common metonym for France. Which is why we have a redirect from one to the other, and in our article on France, it states "It is often referred to as L’Hexagone (The “Hexagon”) because of the geometric shape of its territory." --Jayron32.talk.contribs 21:51, 13 March 2009 (UTC)[reply]


March 12

Born into the Purple

Were all children born to Byzantine Emperors' during their reign styled Porphyrogenitos or Porphyrogenita? Why are Byzantine imperial titles considered honorifcs? No other European titles are called honorific. --Queen Elizabeth II's Little Spy (talk) 03:51, 12 March 2009 (UTC)[reply]

The article you linked has this in the opening paragraph: "However, not every imperial prince or princess was accorded this honorific distinction." Does that answer your question? -- kainaw 03:53, 12 March 2009 (UTC)[reply]
No, I think the not every imperial prince or princess is refering to the Emperor's children born prior to his coronation. --Queen Elizabeth II's Little Spy (talk) 06:18, 12 March 2009 (UTC)[reply]
I think technically they had to be born in a particular purple-painted room in the palace (hence, porphyrogenitos, born in the purple, literally), although the term and the concept were later generalized to any child born to a reigning emperor. The concept was also borrowed by other states, like Jerusalem. I don't think it was a special style they were entitled to, it was just a description; medieval titles were not as formalized as they are now, and while the Byzantine emperor could have a great number of descriptive titles they were not "official" in the sense you seem to be thinking of. There is of course one emperor who is commonly called porphyrogenitos. Adam Bishop (talk) 08:48, 12 March 2009 (UTC)[reply]
And then there was the infamous Purple Rubyred . . . // BL \\ (talk) 21:16, 12 March 2009 (UTC)[reply]

Last person burned at the stake

Who was the last person burned alive at the stake in Europe? And who was the last in Britain? Catherine Murphy was sentenced to it (1789), but in reality, she does not seem to have ben burned alive, so who was it? --85.226.44.201 (talk) 11:57, 12 March 2009 (UTC)[reply]

WP says it was Catherine Murphy in 1798 / London for "coining", ie counterfeiting money. Oops, it seems she had been hanged prior to the execution by burning. Googling indicates one Phoebe Harrius in 1786, who was burned for the same crime. --Cookatoo.ergo.ZooM (talk) 12:16, 12 March 2009 (UTC)[reply]
However, according to Execution by burning#Modern burnings, it is still going on in other parts of the world. Astronaut (talk) 00:21, 13 March 2009 (UTC)[reply]
Yes, people are still burned, but not by law. Several women in Britain were sentenced to be burned in England until 1789, but it seems that in reality, most of them was strangled first. So who was literally burned last? Janet Horne was burned alive in 1727.--85.226.44.201 (talk) 10:04, 13 March 2009 (UTC)[reply]

US president with a criminal record

I and I can guess many have read the trivia that GWB is the first president to take office with a criminal record. But wouldn't the what I only can guess is technical treason by George Washington (and presumably some others after him) against Britain count? chandler · 14:34, 12 March 2009 (UTC)[reply]

So what criminal record has GWB got? And how many presidents got - or nearly got - a criminal record while a president? 89.243.40.164 (talk) 00:34, 13 March 2009 (UTC)[reply]
Was he ever actually convicted in a court? Had the revolutionaries lost, he probably would have been, but they didn't. Generally you only get charged with crimes if you lose a war. --Tango (talk) 14:37, 12 March 2009 (UTC)[reply]
I would have guessed Britain "convicted" every person in the revolutionary army right on the spot, even if they werent able to go through with it so to speak. chandler · 14:49, 12 March 2009 (UTC)[reply]
Yes, but the question asked about a criminal record not commision of a crime which are two different concepts. Generally, people only create files and keep records of crimes you are either charged with or convicted of. Dubya definately has a conviction on record (a 1976 DUI charge that he pled guilty to). Not sure of any others, but claiming that someone committed an action which may or may not have been considered by others to be a crime (like Washington's "treason") is NOT the same as saying that someone had a criminal record. Several other presidents have admitted to actions which were may have been crimes at the time they committed them (Clinton and Obama both admitted to using illicit drugs, for example), however since neither was ever tried and convicted of such, neither has a criminal record. --Jayron32.talk.contribs 14:58, 12 March 2009 (UTC)[reply]
"Convicted" means found guilty by a judge/jury/magistrate. Unless charges were brought against him and there was a trial in which he was found guilty, then there was no conviction. --Tango (talk) 15:26, 12 March 2009 (UTC)[reply]
In the justice systems of yesteryear such procedures as "holding a trail" were perhaps overlooked if they really wanted? ;-) chandler · 15:37, 12 March 2009 (UTC)[reply]
You might be tried in absentia, but there would most likely be a trial. We're not talking that long ago - the UK's legal system was pretty well established. I can't see why they would have tried Washington in absentia - I don't see any point to it. --Tango (talk) 16:16, 12 March 2009 (UTC)[reply]
To be accurate, "criminal record" should be clarified as "adult criminal record in the United States". We do not know about juvenile records or foreign records. -- kainaw 16:39, 12 March 2009 (UTC)[reply]
Besides, we won the war, so we got to decide who was innocent. Wrad (talk) 16:56, 12 March 2009 (UTC)[reply]
Not who's innocent in Britain. chandler · 17:00, 12 March 2009 (UTC)[reply]
We weren't in Britain anymore when we won. We were our own country. We couldn't have cared less about Britain. Wrad (talk) 18:07, 12 March 2009 (UTC)[reply]
Wrad, do you mean to say that you were actually a combatant in the American War of Independence? If so, it seems you ought to be at the top of this list. Malcolm XIV (talk) 00:06, 13 March 2009 (UTC)[reply]
You meet all sorts of people on Wikipedia, don't you? Why they heck would America care who was innocent in Britain back then? "Oh, guys. Britain called. They said we should all be hanged as traitors, so let's get with it. Hancock, you first..." Wrad (talk) 00:27, 13 March 2009 (UTC)[reply]
I saw a cartoon many years ago, while the USSR was still the enemyt of the U.S. in the Cold War. Soviet general, in uniforms heavy with medals, were sittion around a table in the Kremlin, when one announced that they had all been subpoenad by the U.S. House Unamerican Activities Committee (known in the '50s and '60's for finding commies hiding under every bed). A similar issue was an East German spy chief (Markus Wolf) who, after reunification of East and West Germany, was tried for espionage against West Germany [8]. Hardly seems fair, like charging a soldier with terrorism for shooting back when the fort he is in in a war zone is being blasted to bits and all his fellow fighters killed. "Victor's justice" is no justice. Edison (talk) 17:42, 13 March 2009 (UTC)[reply]
All I'm saying is that you should stop using the word "we" to refer to what other people did centuries ago. It wasn't you, it was they. Malcolm XIV (talk) 01:12, 13 March 2009 (UTC)[reply]
I am an American. So were they. We have done a lot of things as Americans and we'll use whatever pronouns we want to. Wrad (talk) 01:27, 13 March 2009 (UTC)[reply]
Calling one's ancestors "we" is less harmful imho than calling one's present rulers "we", at which no one but me seems to bat an eye. —Tamfang (talk) 07:48, 13 March 2009 (UTC)[reply]
If you can cite a source as to Washington being tried in absentia in Great Britian, please do so. Otherwise, please refrain from arguing in circles. Livewireo (talk) 17:54, 12 March 2009 (UTC)[reply]
Chandler, you seem to have a slight misunderstanding about the system of British justice in the 1700s. Take a look at common law and related articles. The British justice system - and the American justice system that developed from it - was fairly well developed by that time. It may have been a bit corrupt and a bit absurd at times, but the criminal procedures we use today, such as the necessity of prosecution, trial, conviction and sentencing, were well established.
This is not to say that a politically minded British judge and jury would not have convicted George Washington in absentia if the issue came up to them. However, it never did. "I would have eaten the donut if it was on my plate" is different to "I did eat the donut". As a matter of historical fact, Washington was never prosecuted, tried, convicted or sentenced. We have articles are all of those terms, too. --PalaceGuard008 (Talk) 23:00, 12 March 2009 (UTC)[reply]
P.S. If you are really intrested in common law legal institutions in the early days, take a look at Coke's Institutes (the book, not the Wikipedia article) and commentaries available at your local academic library. --PalaceGuard008 (Talk) 23:03, 12 March 2009 (UTC)[reply]

Andrew Jackson was closer than Washington. He was actually held as a prisoner by the British (government). But I'm not sure exactly what his legal status was. --JGGardiner (talk) 23:02, 12 March 2009 (UTC)[reply]

If my understanding is correct, Andrew Jackson was a prisoner of war. Prisoners of war are not criminals: they are simply combatants who have been captured by the other side and are taken "out of the game" for the rest of the war. They are protected by customary and treaty-based international law from harm by their captors. In fact, combatants are immune from prosecution for ordinary conduct during the war, e.g. prosecution for "murder" just because they shot someone on the battlefield. A prisoner of war only turns into a criminal if they are then prosecuted for war crimes - which are violations of the laws of war beyond just participating in a war. --PalaceGuard008 (Talk) 23:07, 12 March 2009 (UTC)[reply]
(edit conflict, consequently sort of redundant) The Continental Congress was officially (I believe) declared an illegal assembly, and everyone who was involved 'could' have been tried for treason, even if the situation hadn't thenceforth escalated into open rebellion. But even if individual presidents-to-be had been tried in absentia, which we have no evidence of thus far, their convictions would presumably have been thrown out when the Treaty of Paris was ratified. Article one states that "his Majesty relinquishes all claims to the Government, propriety, and territorial rights of the same, and every part thereof;" it follows that his Majesty would retroactively relinquish the right to label foreign subjects traitors for agreeing with the document he had just signed. So, much as Clinton could have been tried for smoking pot if he'd been caught, Washington could have been tried for treason pre-Treaty, but neither was, so it's a moot point. --Fullobeans (talk) 23:19, 12 March 2009 (UTC)[reply]
George W. Bush was arrested for drunk driving in Maine in September 1976. According to Newsweek, he went to court a month later, pleaded guilty, paid a US$150 fine and lost driving privileges in Maine for two years. As far as I can determine, he was the only U.S. president who had ever been the subject of a police arrest. Pastor Theo (talk) 02:55, 13 March 2009 (UTC)[reply]
Does an impeachment constitute an arrest? Obviously as Andrew Johnson's impeachment failed it isn't a criminal conviction but would it be considered as equivalent to an arrest?
Similarly what about Richard Nixon - never convicted as far as I'm aware though but for a Presidential Pardon he could've been. AllanHainey (talk) 16:47, 13 March 2009 (UTC)[reply]
And if I had a steak for dinner last tuesday, I would have eaten it. Speculating on what might have happened had events turned out different is pointless. Had Ronald Reagan strangled a man in cold blood, he may have been tried for murder and convicted. Had Harry Truman robbed banks, he could have gone to jail for that. But none of those things, nor the ones you described above, actually happened As has been noted by several people, George W. Bush was arrested, indicted, and pled guilty to a crime, so he has a criminal record. No other president appears to have one, as near as anyone here can find. --Jayron32.talk.contribs 21:48, 13 March 2009 (UTC)[reply]
It's more the fact that he was convicted, rather than how he happened to plead. -- JackofOz (talk) 21:56, 13 March 2009 (UTC)[reply]

science is falsifiable, and religion is not

There was a court case (I think at the Supreme Court level, though it may have been a state Supreme Court) where the ruling was something to the idea of the subject, but I cannot recall the name of the case. I also want to think it was in the 1950s, though I could be dead wrong on the dating. A link would be good. :) --Izno (talk) 19:03, 12 March 2009 (UTC)[reply]

You might be thinking of the Scopes Trial. You might also be interested in Kitzmiller v. Dover Area School District. A Quest For Knowledge (talk) 19:17, 12 March 2009 (UTC)[reply]
Ah, it's probably the latter, though if there are others thought of, feel free to include them. Thanks. --Izno (talk) 19:27, 12 March 2009 (UTC)[reply]
The "see also" part of the Kitzmiller article is a nice list of creationist/science trials of note. --98.217.14.211 (talk) 00:48, 13 March 2009 (UTC)[reply]
I found a website off Google that has a list of Supreme Court cases regarding religion here: http://religiousfreedom.lib.virginia.edu/court/Livewireo (talk) 20:16, 12 March 2009 (UTC)[reply]
Right, but almost all of those have nothing to do with science. --98.217.14.211 (talk) 00:49, 13 March 2009 (UTC)[reply]
As a somewhat related topic, you may be interested in reading Rocks of Ages by noted paleantologist and philosopher Stephen Jay Gould. --Jayron32.talk.contribs 01:12, 13 March 2009 (UTC)[reply]

St John the baptist

Did st john the baptist have siblings?--LordGorval (talk) 20:19, 12 March 2009 (UTC)[reply]

Not according to biblical references. John the Baptist was the son of Zechariah and Elizabeth, an older, childless couple. John's conception is presented as a miraculous event and to an extent foreshadows that of Jesus Christ, born to Elizabeth's cousin Mary. - EronTalk 20:26, 12 March 2009 (UTC)[reply]
(For what it's worth, the Blessed Virgin aside, God seems to have preferred granting pregnancies to older childless ladies: see also Hannah, Sarah.) - EronTalk 20:32, 12 March 2009 (UTC)[reply]
Well, those ladies did need a little extra help to get the biological clocks ticking again. :) Pastor Theo (talk) 02:59, 13 March 2009 (UTC)[reply]

Where the Great Pyramids built on grassy fields?

Looking at the Great Pyramids on Google Earth, I can see vegetation in the area between them and the Nile. But of course, the pyramids are surrounded by sandy desert. But was that always the case? When they were being built, was that desert filled with green grasses and papyrus? --70.167.58.6 (talk) 20:32, 12 March 2009 (UTC)[reply]

According to the Sahara article, it has been about as dry and arid as it is today since around 3400 BC, nearly 800 years before the Great Pyramids began construction. The Nile River itself is extremely fertile directly around it, which is why civilization has been able to thrive in such a harsh climate Livewireo (talk) 20:42, 12 March 2009 (UTC)[reply]
Yes, I believe the pyramids have always been surrounded by desert. however, in the 4500 years since the pyramids were built, the path of the river Nile has moved several times. It is only in recent times (less than 200 years?) that the Nile has been constrained between embankments passing through the Cairo urban area. The pyramids were deliberately built on a dry bedrock plateau well above the river and its annual flooding. During their construction, very heavy stones were transported from other parts of Egypt by river and then dragged up a road from the river. When the pharoh died, they were also taken by river to a morturary temple built on the river bank, and then up a causeway to their pyramid. If you look at this diagram you can see the morturary temples and causeways, and deduce how close the river bank was at the time. Astronaut (talk) 00:16, 13 March 2009 (UTC)[reply]
It is doubtful that the Giza site was grassy. As evidence, Thutmose IV (18th Dynasty), when still a prince, supervised the removal of sand that had half buried the Great Sphinx. (The stele he erected recording the event was discovered in modern times when the re-encroaching sand was again cleared away.) B00P (talk) 04:33, 13 March 2009 (UTC)[reply]

Why China, Russia, India and Brazil Don't Admit Refugees?

Why China, Russia, India and Brazil don't admit international refugees into their countries? Why U.S, Canada, U.K., France, Australia, and New Zealand admit a lot of international refugees? Sonic99 (talk) 21:00, 12 March 2009 (UTC)[reply]

There are never simple answers to questions like this. The reasons why countries do something are often complex, just like the reasons why people do things. Some possible factors might include (speaking for Canada here): 1) a feeling that those forced out of their own country deserve a place to live 2) a belief in the value of, and a history of, immigration in general 3) acknowledgment that we are a wealthy country and deserve to share our good fortune in some way. There are of course those who would say that the number of refugees accepted by these countries is tiny compared with the size of the problem, and others might say that these countries have sometimes contributed to the problems that the refugees are running from. DJ Clayworth (talk) 21:52, 12 March 2009 (UTC)[reply]
The BRIC countries are economically a lot less advanced than the other countries (all OECD countries) that you mentioned. Fundamnetally, the issue is one of economics. For these governments, it is difficiult to accept a large number of refugees, who will place further demand on infrastructure, social services and welfare, when their own nationals often live in terrible conditions themselves. It is both economically impractical, and also politically difficult to justify. In addition to economic issues, there are also social, environmental and political issues.
The US, Canada, Australia and New Zealand are all predominantly migrant countries with a history of accepting migrants from diverse backgrounds and of multiculturalism. The UK and France, while not "melting pots" to begin with, also took in large numbers of migrants - and continue to do so - due to conscientious and political reasons when their large overseas empires were dissolved. Neither of these two scenarios are applicable to the BRIC countries.
While they do not accept large numbers of refugees, all four of these countries often faces refugee issues. In India, for example, there is a significant population of Tibetan refugees from China. In China, there are also significant numbers of refugees from North Korea and also from Vietnam. While the North Korean refugees, and the hardline stance taken by the Chinese government towards them, are highly publicised, the Vietnamese refugees are less so. Most of these are Vietnamese citizens or residents who are classified as ethnic Chinese by the Vietnamese government, though often not being able to even speak Chinese, and who escaped persecution in Vietnam in the late 1970s. In total, China accepted 224,000 Vietnamese refugees, with funding assistance from the UNHCR. The UNHCR has over the last 20 years reportedly provided around US$85m in funding for Vietnamese refugees in China, and the Chinese government has provided US$758m over the same time period. --PalaceGuard008 (Talk) 22:18, 12 March 2009 (UTC)[reply]
The question is loaded, the assumption being that China, Russia, India and Brazil don't admit refugees to begin with. As PalaceGuard008 points out, China and India take quite a few (add to those examples a couple of million from Bangladesh, Nepal, Sri Lanka and Burma. Per the apalling City of Joy, half the population of the city of Calcutta are refugees; that's 7 million+ right there). Russia gets its share from Baltic and Central Asian states, but also has a bunch from the far east (e.g. Mongolia). Brazil is -- almost by definition (if not the epitome of it) -- an immigration-friendly country.
So, when exactly did you stop beating your wife? -- Fullstop (talk) 23:26, 12 March 2009 (UTC)[reply]
Ah, oops. Forgot about Brazil, almost the ultimate "melting pot". =P --PalaceGuard008 (Talk) 23:50, 12 March 2009 (UTC)[reply]
Just a reminder that "refugee" is not equivalent to "immigrant", and muddying the waters between the two leads to slopy political discourse (and hateful Daily Mail headlines). Also, past performance is not a guarantee of future performance, or even of what's going on currently. We all know that Brazil has been created by a historic melting pot process; I for one have no idea of their policy re refugees now. Apples and oranges, folks! And for perspective, for some years the country with the largest number of refugees was Pakistan, because Afghanistan was under attack, and millions fled across the border. BrainyBabe (talk) 07:45, 13 March 2009 (UTC)[reply]
We know that, but a history of accepting immigrants probably makes a country well-disposed to accepting refugees. DJ Clayworth (talk) 14:35, 13 March 2009 (UTC)[reply]
Possibly, but the converse is not always true. Scandinavian countries, while having a history of relatively little (economic) immigration, have done a pretty good job with taking in (political) refugees. So it all depends. BrainyBabe (talk) 15:52, 13 March 2009 (UTC)[reply]
A refugee is someone who seeks refuge. Period. This is regardless of the reasons for seeking refuge. And, it always also has an economic dimension. After all, economic survival is a prerequisite to physical survival.
While there is a distinction between a refugee and an emigrant/immigrant, the distinction has nothing per se to do with finances. The defining characteristic of an immigrant is one who arrives in a country to stay. An immigrant can just as well also be a refugee who stays because the reasons to seek refuge will exist forever (as opposed to seeking/receiving refuge only as long as the reasons to seek/receive refuge exist).
As is the case for most countries (all those that don't feel the need to keep people out), there is no practical distinction between refugees in Brazil and immigrants in Brazil. The need to live there, whether temporarily or permanently, was/is driven by the need to survive. In squalor perhaps, but alive. -- Fullstop (talk) 19:50, 13 March 2009 (UTC)[reply]
You are correct that most immigrants intend to stay, at least for some years and often for life, in their country of destination, and that refugees, on the other hand, wish to go home when the situation is safe to do so. (I say most immigrants -- there are whole categories of exceptions. One is as those Hongkongers who, in the 1980s and 1990s, lined up somewhere else safe to go, usually by the landed immigrant and citizenship route, in case the 1997 handover proved too traumatic; about 30% of those "astronaut families" have returned.)
Unfortunately the situation is not as clear-cut with regard to definitions, Fullstop/Period, and as the responses above have not been as rich in internal links and external sources as RefDesk answers sometimes are, I will add what I can here. You say that a "refugee is someone who seeks refuge [...] regardless of the reasons for seeking refuge." Not quite. According to the 1951 Convention Relating to the Status of Refugees (amended 1967), a refugee is:
"A person who owing to a well-founded fear of being persecuted for reasons of race, religion, nationality, membership of a particular social group or political opinion, is outside the country of his nationality and is unable or, owing to such fear, is unwilling to avail himself of the protection of that country; or who, not having a nationality and being outside the country of his former habitual residence as a result of such events, is unable or, owing to such fear, is unwilling to return to it."
In practice, in destination countries such as Britain, the process of being officially recognised as a refugee is a lengthy and arduous one, and a legal limbo intervenes between the individual claiming the right of asylum and the state granting permanent leave to remain, during which time the individual is referred to as an asylum seeker. Their rights may very well be curtailed during this time: they may not be allowed to work, they may be required to live in a certain place, and so on.
Organisations such as United Nations High Commissioner for Refugees have expanded their remit to other "persons of concern" such as internally displaced persons (e.g. those in Darfur and South Sudan, both officially parts of Sudan). NGOs such as the European Council on Refugees and Exiles and the U.S. Committee for Refugees and Immigrants lobby on behalf of people in these situations, and concern themselves with issues such as forced migration and the fate of the displaced person.
Until the twentieth century, mass population movements, after war or natural disaster, were not necessarily treated bureaucratically, and anyway borders were less fixed. The idea of the refugee was of an individual in political exile: Descartes went to the Netherlands, Voltaire to England, Hobbes to France. You contend that "the reasons for seeking refuge [...] always also has an economic dimension"; I would dispute that, as our list of refugees includes those who could have chosen to live according to the laws and customs of their country of origin, but who felt that something more important was at stake: Ayaan Hirsi Ali fled a forced marriage, the parents of Naomi Klein fled conscription, Mehran Karimi Nasseri refused to live under the supervision of a Belgian social worker.
You state that "most countries [...] don't feel the need to keep people out". As I said, I know nothing of the current law and practice in Brazil, but it is my understanding that most countries do feel the need to keep at least some people out; that is one of the main points of border controls, to control what sort and how many people are allowed to enter the country, temporarily and permanently.
I could go on, but I'll stop now, drawing readers' attention to the fact that our article Refugee barely mentions the term "immigration", except within the names of organisations, in the special case of Israel, and with reference to Illegal immigration, whereas our article Immigration, much more sloppily written IMO, mentions "refugees" eight times. For a reasonable list of push and pull factors, see Emigration. BrainyBabe (talk) 23:30, 14 March 2009 (UTC)[reply]

Copla!

Sorry to disappoint anyone whom the title misled into thinking this was a Klingon or Team America question

A (small) number of Wikipedia articles about Spanish music mention "copla", which seems to be a style of singing or of song. The spanish wikipedia has articles es:Copla and es:Copla andaluza (athough the former might just mean "couplet"). Should we (or do we, under another name) have an article about Copla. Mimetic Polyalloy (talk) 22:30, 12 March 2009 (UTC)[reply]

There is some mention of it in the article on flamenco. Of course, the Spanish WP has an entry on it, which could be translated. --Cookatoo.ergo.ZooM (talk) 00:05, 13 March 2009 (UTC)[reply]
The es.wikipedia article seems to say it's neither a style of singing nor of song (well, not quite), but a style of writing poetry which is intended for use as lyrics to popular folk songs. It originated in Spain, where it's still very common, but it's also now widespread in Latin America. The copla generally deals with romantic themes, and may take its topic from a song, local event, or barroom romance. Some of the great Spanish poets (Manuel Machado, Antonio Machado, Luis de Góngora, Rafael Alberti, Íñigo López de Mendoza, marqués de Santillana, Federico García Lorca) have apparently dabbled in coplas, and it was a poetic form they approached with great respect. I don't dare try to figure out what the form consists of, though, since my Spanish is already stretched to the breaking point. --Fullobeans (talk) 07:20, 13 March 2009 (UTC)[reply]

Bernard Madoff

Hi, sorry my English :D. I want to know when the trial against Madoff ends. I mean when he receive his years of imprisonment/sentence: I am sure this info is on the article Madoff investment scandal but it is not available in Spanish. Can you answer me?. Thanks for all, specially for your patience. :D --190.49.107.200 (talk) 22:41, 12 March 2009 (UTC)[reply]

As he pleaded guilty today, the guilt-part of the trial is over. What remains is sentencing, which isn't scheduled to take place until June (but he may strike a bargain with investigators before then). Mimetic Polyalloy (talk) 22:51, 12 March 2009 (UTC)[reply]
If you're thinking "what a long delay", I rather suspect that's deliberately to give him time to help the investigators (and to put some pressure on him to do so). Apparently they've recovered only around 2% of the fund's money so far. Mimetic Polyalloy (talk) 22:55, 12 March 2009 (UTC)[reply]
When Bernard Madoff pled guilty today, the judge announced that sentencing is set for June 16 at 1:30 p.m. The judge also addressed the length of time between plea and sentencing: "Some of the victims may wonder why do we need so much time. Well, the probation department has to prepare a presentence report. By law, the defendant is entitled to 35 days to review the presentence report before sentencing. We also have to give the parties an opportunity to submit written materials."
The charges against Madoff theoretically could produce maximum sentences of 150 years, if each charge received the maximum sentence and the sentences were served consecutively, rather than concurrently. The government has indicated that it does intend to seek the maximum, 150-year sentence.
Government and private civil actions against Madoff will continue after the sentencing date. John M Baker (talk) 00:19, 13 March 2009 (UTC)[reply]
I've never quite understood why sentences aren't proportional. That is, if a person steal $1000 each from a million people, shouldn't their sentence be a million times longer than someone who only steals that much from one person ? It clearly isn't though, as Bernie Madoff with all that money but will probably end up serving something less than life in prison. StuRat (talk) 06:25, 13 March 2009 (UTC)[reply]
Some sentences are proportional—if you crash a plane you can a different murder charge for each and every person—but I don't think white collar crimes are considered that way. --140.247.250.160 (talk) 20:07, 13 March 2009 (UTC)[reply]
As they pointed out on NPR today - while his investors claim to have lost somewhat over $60B - they only originally deposited something like $12B (still a lot of money...but a lot less than $60B). Some of THAT had been paid out of the fund to people who withdrew from it before it all fell apart - so the amount that is actually fraudulant had to have been spent or squirreled away by Madoff has to be much smaller than that. The $60B number comes about because that's what he TOLD people their money had grown to. So the problem is that people are naively assuming that there is something like $50B sitting in a swiss bank account somewhere - when in fact, since he did not invest a single penny of their money, there never was anything like that amount there in the first place. However, there was a claim that Madoff's wife had pulled $15 million out of the company shortly before it all folded...but since that's less than a tenth of a percent of the ORIGINAL money people put in - and an even more microscopic fraction of what they THINK they are owed. It's very sad...particularly given all of the strenuous attempts several people put in to try to get this scheme shut down as long as 5 years ago - only to be ignored by the Fed's. There will always be criminals - and it's the job of government to catch them. As bad as the Bush years seemed to have been on the day he finally left office - the degree of the problems he left us with is still only just beginning to become clear. SteveBaker (talk) 01:01, 13 March 2009 (UTC)[reply]
Steve, as much as I'm one of Bush's biggest critics (and was very surprised the day he made it into office) do you mind indicating how the Bush administration had anything to do with this case or its investigation (or lack there-of)? Just a general indication would suffice, I expect someone else though would demand facts and figures Rfwoolf (talk) 12:32, 13 March 2009 (UTC)[reply]
I think what Steve is getting at is that, in Madoff's case, the SEC received a detailed denunciation of Madoff around 2005 explaining why his operation could only be a Ponzi scheme. The text of that paper is available on the web. This was not acted upon. The speculation is that there was political direction given to the SEC to go easy in investigating Wall Street during booming times. If the denunciation had come from a defrauded investor, the SEC would probably have been legally bound to act, but in this case it was simply a very well-researched paper from an interested Wall Street observer, and the SEC never took action. --Xuxl (talk) 14:04, 13 March 2009 (UTC)[reply]
Yep. More than that actually: Multiple people warned the SEC on multiple occasions. The directive to reduce regulations on investment banks (such as Madoff) was brought in under political pressure by William H. Donaldson - a Bush appointee - and a fellow member of the deeply worrying "Skull and Bones" organization (of which Bush was a member). I suspect that Madoff's activity in SIFMA, NASD and other industry 'self-regulatory' groups - plus the marriage of his niece to a prominant SEC compliance officer made it difficult for the SEC to go after him...but that's speculation. Many people realised this - many reported their concerns to the SEC. Perhaps the most vigorous was Harry Markopolos. He is a forensic financial analyst and he had warned the SEC repeatedly - in writing and in person - telling them that Madoff's claimed gains were "mathematically impossible". He did this as early as 1999 with actual meetings with SEC staff taking place in 2000 and 2001. He wrote and called the SEC many times over the intervening years pointing out that (in words even I can understand) that this simply must be a ponzi scheme. His efforts culminating in a detailed 20+ page report and sent it directly to SEC regulators in 2005 and called them personally to make sure they'd read it. It simply shouldn't be possible to avoid getting caught in a 12 BILLION dollar fraud with that amount of clear proof. How could anyone imagine that with numbers that big that someone at the SEC wouldn't have looked just a teeny bit deeper? What would it take to stop that kind of investigation? SteveBaker (talk) 03:51, 15 March 2009 (UTC)[reply]
I wonder if any charges can be brought against the SEC and the officers who made those decisions. There has to be some way to punish those orgs that simply choose not to perform the jobs they are mandated to do. StuRat (talk) 00:09, 14 March 2009 (UTC)[reply]
I would hope there would be some sort of investigation. We could perhaps forgive them from not looking deeper in the absence of evidence - but they'd had good evidence 10 YEARS before the shit hit the fan. Someone at the SEC (probably William Donaldson) should be in deep poop over this. Markopolos claims that he figured out what Madoff was up to with just four hours of work and using only publically available information. His boss had asked him to figure out how Madoff was doing what he did - and he found it was 'mathematically impossible' for Madoff to be doing that well for his investors. If one guy can figure it out without knowing in just four hours - how can the SEC claim to have been unable to figure it out after someone told them where to look? Meaghan Cheung (who was head of the New York city office of the SEC) was a recipient of Markopolis' 20 page report - yet she actually DROPPED an investigation of Madoff in November 2007 - two years after she had clear evidence of a Ponzi scheme in progress. Madoff actually sat on SEC advisory committees - and for a while was chairman of NASDAQ...he knew all the right people. There are lots of other stinky dealings - SEC officials with money invested with Madoff who pulled out before the crash hit - despite the huge returns they were getting. Just follow the references from the articles I linked to above...the conclusion is inescapable. SteveBaker (talk) 03:51, 15 March 2009 (UTC)[reply]

March 13

Love of the flag

An earlier question got me thinking. On trips to the USA I've noticed the Stars and stripes is very much in evidence, hanging from gigantic flagpoles, outside shopping malls, in school classrooms, and adorning peoples homes. Is there any other country where you see the national flag so much? Astronaut (talk) 00:35, 13 March 2009 (UTC)[reply]

Texas :) Wrad (talk) 00:37, 13 March 2009 (UTC)[reply]
I wonder if George W Bush flies the Lone Star flag or the Stars and Stripes outside his new place? Astronaut (talk) 00:57, 13 March 2009 (UTC)[reply]
I've noticed that in parts of Europe, for example Bavaria, regional flags are very much in evidence, often a lot more than national flags. --PalaceGuard008 (Talk) 00:39, 13 March 2009 (UTC)[reply]
I never really though about it, but you are right. I lived in Neu-Ulm and I'm sure that the blue and white Bavarian flag was much more evident than the German tricolor. --—— Gadget850 (Ed) talk - 16:17, 13 March 2009 (UTC)[reply]
Denmark in particular, and Scandinavia in general are famed for their flag flying --Saalstin (talk) 00:59, 13 March 2009 (UTC)[reply]
Australia is a particular propagandist, flag-flying state and I've noticed they tend to use their flag a great deal. Dick Smith products used to (and possibly still do) use the Australian flag on all their products which got them into trouble. Many products advertised on TV seem to lead their reasons for buying the product as being "because it's Australian" (and I'm not just talking about the "Australian made and owned" campaign which decorates 99.999% of Australian supermarket products (in part because getting foreign products onto Australian shelves is nearly impossible as I've tried this myself)))). I saw adverts on TV campaigning Australians to get involved in eco-volunteerism invoke certain propagandist elements such as the Australian flag, close-ups of fists with soil (a propagandist element for "taking action") -- and these are just the ones I've seen. In fact it seems that if the Australian government wants the public to do anything, all they need to do is show the flag, and say that not doing what they want would be unAustralian. And lastly in case anyone disagrees with what I've said, I'm sure that nobody would argue with the fact that Australians are a particularly proud bunch. :) Rfwoolf (talk) 09:42, 13 March 2009 (UTC)[reply]
Oh, yes, proud all right. But the flag's an interesting issue. There have always been flags on buildings etc, but flying them in one's front garden was rarely (virtually never) seen until the last 20 years or so. That seems to have coincided with a number of debates about its design (the inclusion of the Union Jack in the top left corner is the main sticking point*), and there have been various public competitions sponsored by major newspapers, seeking a design that all Australians (well, you'll never please everyone) can be happy with. These have also coincided with a lot of debate about republic vs. monarchy. But all they've succeeded in doing so far is entrenching the current design. The Flags Act 1953 was amended so that it now requires a referendum to change the design, not merely an act of parliament. But even for those who do not particularly like the current design (such as me), they're a lot happier about flying whatever the current flag is in their front yard (me again) than they would have been 20-25 years ago. That in itself is a positive development, imo. (*Another objection is that it closely resembles the NZ flag, and they're often confused. But we had ours first, so if any change to either of these flags is going to happen for that reason alone, the New Zealanders should be changing theirs.) -- JackofOz (talk) 21:47, 13 March 2009 (UTC)[reply]
That's some interesting anectodal history about the flag, Jack. However, you disappoint: I had run for cover in anticipation of the Australian brigaide (sic), only to be fed a calm yet surprising explanation on pride and the Australian flag. Cheers. Rfwoolf (talk) 22:32, 13 March 2009 (UTC)[reply]
In recent years, the Australian flag has been co-opted by xenophobic, almost racist groups, especially in areas of low socio-economic standing in the coastal cities (see, e.g. 2005 Cronulla riots). This has led to concerns about what it is doing to the image and meaning of the flag. --PalaceGuard008 (Talk) 00:34, 14 March 2009 (UTC)[reply]
Most people realise that racist hoons can associate their flag with whatever racist atrocities they choose to commit, but all that succeeds in doing is even further lowering the reputation of the hoons in the eyes of the public, and doesn't have any implications for the place of honour a national flag has. I understand the concerns some people have, but I've seen no evidence that the flag is less respected now than it was before the riots. -- JackofOz (talk) 03:35, 15 March 2009 (UTC)[reply]

I live in a country that is probably on the opposite end of the spectrum from the U.S. and Scandinavia. In Poland, the national flag is generally flown outside private buildings only on special occasions: national holidays, national mournings, big sporting events, etc. Until a few years ago it was actually illegal for the regular citizen to fly the flag except on a national holiday. It's legal now, but the attitude is changing slowly. Some people may be afraid that such overt display of patriotism would make them look too jingoistic. Some others think that if the flag is flown everyday, it will become mundane and will lose its special, festive meaning. So it may be that less flag-flying doesn't really mean less love of the flag and of the nation it represents. Its just the notion that special symbols are for special occasions. — Kpalion(talk) 11:30, 14 March 2009 (UTC)[reply]

There's a pretty high number of flags per square km in Oslo, on the 17th of May. --NorwegianBlue talk 15:53, 14 March 2009 (UTC)[reply]

"My vote won't make a difference" mentality

Do we have an article that covers how, many voters feel that their single vote won't make a difference so they don't vote, but probably hundreds of thousands of other people feel the exact same way, and if each of those people did vote it would mean the difference of hundreds of thousands of votes?
I'm going be sending an email to many of our South African clients living abroad, trying to convince them to participate in the upcoming South African election, and a reference to this issue would be very helpful. (It has just become legal for South Africans living abroad to be able to vote) Rfwoolf (talk) 08:59, 13 March 2009 (UTC)[reply]

I think Voter turnout and Paradox of voting might have some of what you're looking for. --JGGardiner (talk) 09:12, 13 March 2009 (UTC)[reply]
Paradox of voting seems quite relevant, if not a bit complex. It also doesn't seem to cover the fact that in proportional electoral systems, such as South Africa's, it is not simply about which party wins, it is more about how many seats in parliament a particular party receives, in other words it's not just about "who wins" but is also about "by how much do they win". Again, if the current ruling part in South Africa (the African National Congress receives more than two-thirds of the votes, they will be able to unilaterally amend the constitution -- "it's not about who wins, but by how"! Rfwoolf (talk) 09:23, 13 March 2009 (UTC)[reply]
Even then, it seems unlikely that a single vote would result in one more seat for a given party. You have to get down to elections with only a few dozen participants for it to be likely that a single vote will affect the outcome. StuRat (talk) 00:04, 14 March 2009 (UTC)[reply]
rational ignorance may be relevant too. —Tamfang (talk) 20:52, 13 March 2009 (UTC)[reply]
I particularly enjoyed reading rational ignorance, so thanks for the reference. Although it is relevant I don't find it quite as apt as Paradox of voting, but again, thanks for mentioning it. Rfwoolf (talk) 22:30, 13 March 2009 (UTC)[reply]

Court Paintings

Why judicial court proceedings are painted and not photographed? Slmking (talk) 11:16, 13 March 2009 (UTC) slmking[reply]

They aren't always, but when cameras are banned from the courtroom -- as they often are -- the only other way to get images from the proceedings is to have someone draw or paint them. Here's an interesting article on three British courtroom artists. -- Captain Disdain (talk) 11:27, 13 March 2009 (UTC)[reply]
FYI, they're usually sketched with some kind of colored pencil or pastels, rather than painted. --Sean 16:35, 13 March 2009 (UTC)[reply]

Anti-discrimination laws in the US

If a company doesn't hire employees from, for example, Japan I suppose it is considered racism. However, what if the company doesn't buy cars from, for example, Japan. is it also a form of racism? What if the company doesn't buy services from a specific country or region? Does the company need to have a policy against buying services or product from a specific country to be considered racist?--80.58.205.37 (talk) 12:59, 13 March 2009 (UTC)[reply]

I am not a laywer. Not even close. But I think you're confusing Racism with Nationalism. In any case, what would be the point of a law that protects foreign businesses? Equal opportunity laws and policies exist to protect citizens of your country from discrimination. APL (talk) 13:10, 13 March 2009 (UTC)[reply]
Not too long ago, not buying from a particular country was considered a shining example of anti-racism. Self-imposed South African embargoes were quite the rage before Mandela came to power. Rmhermen (talk) 13:14, 13 March 2009 (UTC)[reply]

Depending on the justification that you give internally in the company you could be considered racist. If you don't buy products from South Africa because you don't agree with their government, it is a political opinion. If you don't buy African products because you think the quality is low, it is a commercial decision. It is clearly not enough to decide against products or services of some origin to be racist. You have also to infringe some right of someone - a right to be treated with respect, or be judged by the same standards. Anti-discriminatory laws are normally directed towards protecting members of the society (American citizens or holders of a Green Card) . There is a thorough official list of prohibited personnel practices in the US here--Mr.K. (talk) 13:25, 13 March 2009 (UTC)[reply]

But what about the business owned by a war veteran, which won't buy products or services from the country where the owner lost his legs? (I know that is a ficticious generic example, but it is based on a late friend of my mother's who would not buy Japanese products because he was interned in a Japanese POW camp during WWII) I think you will have a hard time convincing such people to change their views. Astronaut (talk) 14:33, 13 March 2009 (UTC)[reply]
Pet peeve #231: interred=buried; interned=locked up, e.g. in a POW camp. DJ Clayworth (talk) 15:58, 13 March 2009 (UTC)[reply]
Typo corrected Astronaut (talk) 16:35, 13 March 2009 (UTC)[reply]
Is the term you're looking for here boycotting? Rfwoolf (talk) 15:53, 13 March 2009 (UTC)[reply]
Boycotting—a practice whose rationale is not purchasing products from a supplier whose policies are objectionable—is a matter of the consumer's choice (whether wholesale or retail) and not regulated by legislation. Compare that with the original case: denying employment to an (otherwise well-qualified) applicant based on race, religion, gender, sexual orientation, etc. would be discrimination and a violation of laws that exist to protect against this. -- Deborahjay (talk) 16:35, 13 March 2009 (UTC)[reply]
First of all you're mistaken about the 'original case' if you re-read the OP's question (he specifically excludes not employing someone by race but goes on to supplies and suppliers). Second, I refer to boycotting in its English form and not the encyclopedia article, which says "A boycott, an organized ostracism as a means of protest" -- while generally that may refer to consumers, the English definition could still apply here generally. It's just that, to read all these posts describing something that sounds a lot like boycotting (such as 'embargoes') and yet never seeing the word 'boycott' seemed weird to me. Finally, I agree that not employing someone based on race would be prohibited by legislation, but in the case of overseas suppliers I think it's a bit more cloudy, I mean perhaps there are laws that might touch upon it such as fair trade laws. Moreover the OP asked whether it would be considered 'racist', not whether it would be considered illegal or unlawful. "Racism" is more of a philosophical term but in mainstream it refers to racial descrimation, which I guess would apply to a company that descriminates amongst its suppliers based solely upon race, but again, whether this is unlawful or not is another matter -- I do suppose the act of descrimination itself would not be unlawful, but going around to your employees with memos that state "only use suppliers in predominantly caucasian countries / non-asian countries / non-Chink countries / non-Nigger countries" could probably land them in some hot water Rfwoolf (talk) 17:43, 13 March 2009 (UTC)[reply]

Making an area attractive to graduates/professionals increases its job creation - looking for research/articles

I remember reading some research somewhere that the way to increase employment in an area is to get professional people to want to live there, because they are the type of people who create jobs. Can anyone point out any articles or research papers about this, particularly any relating to the UK please? Thanks. 89.242.87.233 (talk) 15:57, 13 March 2009 (UTC)[reply]

This American article may be of interest; there's no reason to suppose things should be terribly different anywhere in the developed world. The never-opinion-free Paul Graham opines about how to build a new Silicon Valley here (although I think he downplays the role of money and finance). 87.115.143.223 (talk) 18:36, 13 March 2009 (UTC)[reply]

I'm looking some research that backs up the kinds of claims made in for example the latter half of this article, http://www.strom.clemson.edu/teams/ced/econ/2-2No5.pdf although I'm not very interested in education or airports. Thanks. 78.146.245.212 (talk) 00:19, 14 March 2009 (UTC)[reply]

Polish reporter case

I wonder if the polish government have done anything at all after a pole reporter was captured and taken hostage in Pakistan and was "probably" beheaded(sorry if i am wrong).like daniel pearl. is there somebody doing something to apprehend the people responsible?? —Preceding unsigned comment added by 61.95.140.188 (talk) 18:33, 13 March 2009 (UTC)[reply]

That would be Piotr Stańczak, whose government's actions are discussed briefly in the Piotr Stańczak article. --Fullobeans (talk) 19:27, 13 March 2009 (UTC)[reply]
Pakistan is engaged in what is virtually a civil war; it has a huge number of people that I'm sure it'd like to find and bring to justice, it clearly isn't succeeding. I'm not sure pressure from Poland, or anywhere else, could realistically persuade it to do anything it isn't already doing. 87.115.143.223 (talk) 21:30, 13 March 2009 (UTC)[reply]
Pakistan needs to purge the ISI of it's Taliban supporters, but they may not be willing or able to do so. Thus, the nation may be doomed to collapse. StuRat (talk) 23:59, 13 March 2009 (UTC)[reply]
So, first of all, not a reporter, but a geologist looking for oil. You may have confused Stańczak with Waldemar Milewicz, a Polish reporter killed in Iraq in 2004. The government has been accused of some inactivity to save Stańczak and then to capture his murderers – but then, the current government is being accused of inactivity in general (the latest story is of Prime Minister Donald Tusk and some of his ministers playing football instead of voting on pension reform bills in the parliament). But it could also be that the government is doing something, but governments usually don't like to talk much about the details of what their secret services do. Poland has also blamed the Pakistani government for not being coöperative enough. — Kpalion(talk) 11:52, 14 March 2009 (UTC)[reply]

Why is Descartes' daemon evil?

How did Descartes know that his evil daemon, if it existed, was evil? Is this judgement still universally accepted? NeonMerlin 21:21, 13 March 2009 (UTC)[reply]

I'm not sure Descartes was talking about a real "demon" with horns and a pitchfork who really existed, rather he was speaking in terms of a "philosophical construct" designed to help us understand something of the nature of humanity. I mean Plato didn't really believe that there was a cave somewhere with people tied up watching shadows on the wall, nor did Rousseau really think that everyone signed a document where we gave up our absolute freedom in exchange for security. Like those examples, Descartes' "Evil Demon" was meant to be demonstrative, not accepted as a literal fact... --Jayron32.talk.contribs 21:39, 13 March 2009 (UTC)[reply]
Descartes was positing something that was causing himself to be entirely deceived, such that his senses did not correspond at all with reality. It's 'evilness' was not part of his argument; it was just a name he gave this idea (pretty reasonably if it's sole purposes was to deceive Descartes). DJ Clayworth (talk) 21:56, 13 March 2009 (UTC)[reply]
Well, what if the daemon was just mistaken, and was trying to correct for what he or she considered to be Descartes own misconceptions? Then it would just be an erroneous, but good-intentioned, daemon! ---98.217.14.211 (talk) 18:24, 14 March 2009 (UTC)[reply]

Delving into matters dualistic is devilishly exasperating and the terra firma from where one posits oftentime takes on ridiculous content and form isolating oneself from reality. Yet what is reality? Descartes knew his Daemon. Whether he actually thought it was evil is a conjecture and as such it must remain. —Preceding unsigned comment added by 125.18.237.2 (talk) 13:09, 15 March 2009 (UTC)[reply]

Ass's paradox

What is the name of the philosophical dilemma which involves a rational ass having to choose between two identical bales of hay? I remember reading about it in a book by Peter Cave (the pop philosopher). Any help would be appreciated --67.43.158.218 (talk) 22:54, 13 March 2009 (UTC)[reply]

...that are equally far from the ass, which starves to death rather than make an "irrational" choice, yes?--Wetman (talk) 23:11, 13 March 2009 (UTC)[reply]
Buridan's ass - Nunh-huh 23:12, 13 March 2009 (UTC)[reply]
It seems odd that they ascribe such characteristics to an animal, which I would call anthropomorphization. Humans are the ones who are often paralyzed by indecision, say by forming endless committees to study problems until the problems become insurmountable. StuRat (talk) 23:54, 13 March 2009 (UTC)[reply]
It makes sense to me. The idea is that the ass isn't smart enough to form the abstract idea that either choice will do, whereas the human observer is. (Not to say that asses are really that asinine or that humans are really that smart, but it's a story. And each species does have its moments.) --Anonymous, 07:03 UTC, March 14, 2009.
But it isn't a lack of intelligence which makes one indecisive, intelligence does that. A stupid animal can make a random choice (it might just go with whichever one it saw first), while a person would want to weigh the pros and cons of each choice first. StuRat (talk) 04:55, 15 March 2009 (UTC)[reply]

Whoever "thought" it up has never observed a donkey closely, it would eat bothhotclaws 14:01, 14 March 2009 (UTC)[reply]

I'm surprised we don't have an article on the monkey trap, which works on a similar principle in that the monkey remains trapped rather than let go of the bait.--Shantavira|feed me 19:55, 14 March 2009 (UTC)[reply]
That is often used as an example of greed, but I do think it's really a lack of intelligence. That is, the monkey doesn't realize that letting go of the banana will allow his hand to slide back out of the jar. StuRat (talk) 04:58, 15 March 2009 (UTC)[reply]
As for committees, they're comprised of people who individually can do nothing, but collectively decide that nothing can be done (not original). Anyone who wants to actually do something bypasses committees and just does whatever it takes. -- JackofOz (talk) 21:33, 14 March 2009 (UTC)[reply]
Donkeys are not stupid. Robert Louis Stevenson went on a camping trip with Modestine, and wrote Travels with a Donkey. He summed up her character as "Her faults were those of her race and sex; her virtues were her own." BrainyBabe (talk) 09:21, 15 March 2009 (UTC)[reply]

March 14

G20 members

Why do the United Kingdom, France, Germany and Italy get their own seats on the G20, even though they are also members of the European Union? Who then was a gentleman? (talk) 06:38, 14 March 2009 (UTC)[reply]

I would think it's because, while they are part of the EU and thus sub-portions of a greater economic power, the countries themselves still operate as individual economies and are, in themselves, major economic powers even without taking into consideration their membership in the union. Master&Expert (Talk) 06:57, 14 March 2009 (UTC)[reply]
I think our article on the topic puts it clearly enough in its first paragraph: "The G-20 (more formally, the Group of Twenty Finance Ministers and Central Bank Governors) is a group of finance ministers and central bank governors from 20 economies: 19 of the world's largest national economies, plus the European Union (EU)." The United Kingdom, France, Germany and Italy are among the 19 world's largest national economies. The European Union's seat is separate from them. -- Captain Disdain (talk) 10:55, 14 March 2009 (UTC)[reply]

steve jobs' Apple salary

a question about Steve Jobs' leave from Apple starting in December (he is to return in June). What happens to his $1 salary? Does he keep it at a prorated rate, so for example if he doesn't work from January to June (6 months) he only gets paid 50 cents for that year, or what? Thanks! —Preceding unsigned comment added by 94.27.159.188 (talk) 14:50, 14 March 2009 (UTC)[reply]

St Zossima/ Sossimo?

Called patron saint of Ukrainian beekeepers. Went to Egypt to bring back wax for the church. What is known of his biography and/or details of his legends? When did he live? Is he the same as monk Zossimo, of a monastery in the far north of Russia? How did he become involved with the penitent saint Margaret in the Middle East?Salkeld (talk) 18:18, 14 March 2009 (UTC)[reply]

It looks like there were a whole bunch of Zosimos. I'm not sure which is the one you're asking about, but that's at least a good place to start. --Fullobeans (talk) 22:43, 14 March 2009 (UTC)[reply]
The patrons of Russian bees and bee-keepers are Zosimus (April 17) and Sabbatius of Solovki. Your "penitent saint Margaret" is i think Mary of Egypt—that would be Zosimas of Palestine.—eric 05:49, 15 March 2009 (UTC)[reply]

is international law about "morality"?

is international law about "morality" or is it just procedural? Specifically I'm interested in the International Court of Justice...but that article doesn't even include the word moral/morals or ethic/ethics/ethical except that judges should be of high moral character. —Preceding unsigned comment added by 94.27.159.188 (talk) 19:08, 14 March 2009 (UTC)[reply]

Is any law about morality? "Murder is wrong" is a moral statement. "Murder is against the law" is a legal statement. You could argue that murder is against the law because it's immoral, or you could argue that murder is against the law because it's impossible for society to function if people are running around killing each other with no repercussions. In secular nations, most laws exist for practical reasons, to keep society functioning smoothly. That's why we have the term "morality laws:" most laws are not based in morality alone, though morality may give support to the law. For those of us who aren't legal scholars, though, the line is often blurry, and we imbue laws with a moral authority as well as a legal authority. Additionally, when someone is suspected of a crime, his sense of morality may be called into question during his trial, or by a judge during his sentencing. A person with no apparent concept of morality is easily painted as a threat to society, whereas a person with a strong moral compass might be seen as a good person who made a mistake and will be easy to rehabilitate. Technically, a person should not be convicted on the basis of immorality alone, but juries can be influenced by a skillful character assassination.
So, to answer your question: international law is just that. Law. Not morality. It exists to discourage nations from aggravating one another constantly, from invading one another indiscriminately, from wiping one another out completely, from massacring civilians, from killing prisoners, and other fun stuff. There are good, practical reasons for all of these laws, but what brought most of them into being was the moral outrage people felt during certain events of the 20th century. That outrage is expressed again every time a similar act or event pops up in the news, and international governing bodies rarely do anything to discourage it. Belief in the morality of law, theoretically, encourages people to obey the law; similarly, belief in the immorality of genocide (theoretically) discourages people from supporting figures who advocate genocide. Presumably that's why we so often hear heavy, philosophical terms like "crimes against humanity" being bandied about. When it comes down to it, though, the International Court of Justice conducts its trials on the basis of laws and legal precedents, not the moral sentiments of its members or of the public. Those sentiments may motivate them or influence their decisions, though; again, it's a blurry line. And there is, of course, the issue of who goes to trial and who gets a free pass. But the laws themselves, even the ones for contentious issues such as war crimes, are fairly dispassionate and unambiguous and capable of being implemented in a purely procedural manner. You might be able to learn more at the ICC website. --Fullobeans (talk) 03:56, 15 March 2009 (UTC)[reply]

What happened to insurance for falls in real-estate prices?

"On July 30, 2002, Deborah Woods, a dental hygienist in Syracuse, New York, became the first person to buy protection against a market decline in the value of her home." says http://www.fastcompany.com/bookclub/excerpts/1591391539.html Another article here: http://www.forbes.com/2002/08/28/0829whynot.html The company concerned no longer seems to have a functioning website: www.realliquidity.com so it has probably gone out of business. What are the details about this please? Just how soon did the insurer go bust (I assume) and did people lose their insurance premiums? Thanks. 89.242.127.126 (talk) 20:03, 14 March 2009 (UTC)[reply]

I don't know the specific details here, but such an insurance scheme would seem likely to fail whenever there is a significant general real estate market decline. Another insurance branch with a similar problem is disaster insurance. The problem is that when a major disaster hits, the companies which offer such policies go bankrupt. There the solution has been for the government to re-insure the insurance companies. Of course, this could only work if the disaster or market decline is of such a scale that the government can handle it. StuRat (talk) 04:50, 15 March 2009 (UTC)[reply]

Holocaust implementation

My question really concerns identifying Jewish people. How did the nazi's know which people at which addresses were Jewish. I know in some cases local peasants "ratted" the jews out. However, what I don't understand is how did the nazi's, if their concepts were based on race, identify secular jews or converted jews? Did they check the synagogue archives or something. So ye, my question is how did the nazi's know who to kill. --Thanks, Hadseys 23:16, 14 March 2009 (UTC)[reply]

they didn't differentiate between secular and converted jews or even quarter jewish people etc. in fact a lot of hitler's closest advisers failed the strict requirements (were too jewish by ancestry) and had to have papers forged that their grandmother cheated on their grandfather (the jewish person in their family) with a non-jew, and that they are the result of this coupling. lots and lots of officers in the third reich got papers of this kind. —Preceding unsigned comment added by 94.27.159.188 (talk) 23:28, 14 March 2009 (UTC)[reply]
The only synagogue in Vienna to survive Kristallnacht was the Stadttempel. One reason cited is that its situation made torching it too much of a fire risk to surrounding (i.e. Christian-owned) buildings. Another reason is that it served as the community offices, and so all the birth, marriage, and death certificates were there, with names and addresses. BrainyBabe (talk) 23:44, 14 March 2009 (UTC)[reply]
Also, what peasants ratted any Jews out? All the Jews lived in the cities. Adam Bishop (talk) 01:30, 15 March 2009 (UTC)[reply]
Heck no! I mostly know about Poland, which may or may not be representative, but, to quote our History of the Jews in Poland article, "there were probably 3,474,000 Jews in Poland as of September 1, 1939 (approximately 10% of the total population) primarily centered in large and smaller cities: 77% lived in cities and 23% in the villages. They made up about 50%, and in some cases even 70% of the population of smaller towns, especially in Eastern Poland." Sure, 77% is the majority, but it's by no means "all." --Fullobeans (talk) 02:07, 15 March 2009 (UTC)[reply]
Oh, I was thinking of Germany. Nevermind. Adam Bishop (talk) 07:36, 15 March 2009 (UTC)[reply]
Just a reminder, the OP's question is still unanswered. Dismas|(talk) 02:30, 15 March 2009 (UTC)[reply]

Found something! This link has some good, though unsourced, answers to this question. Some of the points made are: tracking down every Jewish person in Germany was a long, gradual process, and the burden of proof lay with the person under suspicion of being Jewish (or part Jewish). Modern living tends to produce a lot of paperwork: marriage certificates, birth certificates, baptism certificates, death certificates, military records, charitable donation records, court papers, etc., all of which may either imply your religion or state it directly. Generally, you are given a copy of these documents, and another is kept on file by the office that issued it. So even if you were a completely assimilated, secular Jew who never told anyone about your ancestry and had never been circumsised or sent to Hebrew school, could the same be said for your parents? For their parents? And could you prove this if asked? No? Highly suspicious; you must be hiding something. Even if the government couldn't track down documents condemning you, I'd imagine you'd also be in a pickle if they failed to track down documents exonerating you. And, from Googling around, it looks like everyone had to produce proof of ancestry ("ariernachweis") in order to do stuff like own a shop or attend university. --Fullobeans (talk) 02:53, 15 March 2009 (UTC)[reply]

I doubt the German police had the resources to look at any of that. For a country that carried out such a large-scale genocide, the Gestapo was extremely limited in its power to infiltrate society. Take a look at Gestapo#Daily_operations. I quote: "Contrary to the popular belief, the Gestapo was not an omnipotent agency that had agents in every nook and cranny of German society. [...] The District Office in Nuremberg, which had the responsibility for all of northern Bavaria employed a total of 80-100 informers in the years 1943-1945.[2] The Gestapo office in Saarbrücken had at its service 50 informers in 1939."
The article goes on to say--and this answers the OP's question--"Thus, it was ordinary Germans by their willingness to denounce one another who supplied the Gestapo with the information that determined who the Gestapo arrested." Hardly surprising, considering that it was the German public who put Hitler in power. --Bowlhover (talk) 03:38, 15 March 2009 (UTC)[reply]
Interesting statistic! That being said, it wouldn't be too much work to phone or write the church where a person claimed to have been baptized, the city hall where they claimed their mother was born, etc. Clerks, officials, clergy, and hospital employees were also part of the collaborating public. If the individual under investigation was, in fact, solely responsible for their own exoneration, though, perhaps the investigators didn't even bother making phone calls, or only did so in high-profile cases. --Fullobeans (talk) 04:15, 15 March 2009 (UTC)[reply]
Addendum to Fullobeans post: There was a document, called the "Ariernachweis" (the de WP has an article on it). This was a collection of the birth and marriage documents of the relevant person, their parents and grandparents. This documentation HAD to be supplied by ALL German citizen in employment. Absence of the Ariernachweis led to dismissal and, after 1941, was the basis of deportation, ghettoisation and murder in the Holocaust. PS: I will have a look at translating the article, as it only exists in German and Russian. --Cookatoo.ergo.ZooM (talk) 09:37, 15 March 2009 (UTC)[reply]

March 15

Siberia Independence

Why Siberia or part of Siberia didn't declare itself an independent country when the Soviet Union collapsed? Kazakhstan declared itself an independent country even though it had a large slavic population. 72.136.108.97 (talk) 02:29, 15 March 2009 (UTC)[reply]

Kazakhstan and the other countries that declared independence were already somewhat independent, in the sense that the USSR was considered to be exactly what its name suggests, a union of republics. See Republics of the Soviet Union. Siberia was not one of those republics, all of Siberia was part of the Russian Soviet Federative Socialist Republic, which is now Russia, the part that was left after the others broke away from the USSR. The RSFSR had, and now Russia has, numerous autonomous provinces within it, including many in Siberia, but it would be difficult for sparsely populated regions of Siberia to declare independence. The ones that have a majority non-Russian population don't have the resources to survive independently, and the ones that are majority-Russian wouldn't have any interest in being independent anyway. In any case, Russia is not interested in letting any of them go (Chechnya for example). Adam Bishop (talk) 03:27, 15 March 2009 (UTC)[reply]
To emphasise a point Adam Bishop made in passing: Siberia is a geographical term. It never appears to have existed as an administrative entity at any level (still less a 'national' entity). --ColinFine (talk) 09:23, 15 March 2009 (UTC)[reply]

What is a "sharuck"?

I'm teaching myself a song, probably from sometime between 1910-1930, that has a line that goes: "I could tell you a tale of a great white whale or a sharuck who played at a fiddle . . . ." Obviously the whale refers to "Moby Dick," but does anyone have any idea what a "sharuck" is or what the singer might be referring to? Thanks. Crypticfirefly (talk) 03:46, 15 March 2009 (UTC)[reply]

Are you sure it's not "shark"? --Fullobeans (talk) 04:59, 15 March 2009 (UTC)[reply]
I'm absolutely sure it isn't "shark"-- the song is in a printed book. But I'm keeping an open mind, can you think of a fiddle-playing shark story? Crypticfirefly (talk) 05:08, 15 March 2009 (UTC)[reply]
Hmm. Could it be a bird? Myna bird, to be exact. Although the ones I've met only played the bass viola. --Fullobeans (talk) 05:36, 15 March 2009 (UTC)[reply]
If you don't get an answer here by the time this is archived in a few days, try reposting on the language desk. BrainyBabe (talk) 09:24, 15 March 2009 (UTC)[reply]

Empire of Greece

What would have been the world's (especially Western Europe) reactions have been if the newly Greek state had decided in 1832 they wanted to be the Empire of Greece instead of the Kingdom of Greece? The main reason for the establishment of a Greek monarchy, beside Greek independence, was to restore the Byzantine Empire; even the candidates has to be descendants of the past emperors. Western Europe seem reluctant to recognize emperors, ie. the Russian Emperors weren't recognized by all of Europe as an empire at first. --Queen Elizabeth II's Little Spy (talk) 07:36, 14 March 2009 (UTC)[reply]

Considering that the Greece of 1832 barely extended beyond the Peloponese (see the dark blue area on this map), calling itself an "empire" would certainly have been seen as rather pretentious by many -- and it could have offended the Russians. AnonMoos (talk) 12:34, 14 March 2009 (UTC)[reply]
Indeed. Besides this, profiling itself as a direct successor to the multi-ethnic medieval empire rather than as a Greek nation-state would also have carried with it certain territorial implications. It would theoretically have left the door open for expansion into the territories of other nations. Iblardi (talk) 12:41, 14 March 2009 (UTC)[reply]
Greece of 1832 was as big as any of the former medieval empires, ie. Empire of Trebizond, Empire of Nicaea and a little smaller than the Latin Empire. I not talking about the Empire in Justinian's days. Technically by the 1400s the empire only consisted of Greece and some part of the Turkish coast. The door of expansion only leads into Ottoman Turkey which almost all of Europe was taking advantage of and probably some Slavic Balkan countries. --Queen Elizabeth II's Little Spy (talk) 13:20, 14 March 2009 (UTC)[reply]
All those medieval states either were in control of Constantinople, had been in control of Constantinople, or had intentions to (re)gain control of Constantinople in the not-too-distant future. Greece in 1832 had only achieved independence as a result of support from European countries, and couldn't afford to offend those countries. At that point Greece most needed to quietly consolidate its nationhood for a while, in preparation for future challenges, instead of issuing provocatively public irredentist threats against the Ottomans. Also, declaring Greece to be an empire would have meant that the junior cadet scion of the house of Wittelsbach (Bavaria) was declaring himself to outrank the senior head of his dynasty, which would have been an overturn of the prevailing royal protocols of the time. The idea doesn't make too much sense from most points of view... AnonMoos (talk) 05:29, 15 March 2009 (UTC)[reply]
That may already have been bad enough in an age of emerging nationalism. Besides, if the Greeks had profiled themselves as successors to the (unpopular) Byzantine empire, they would probably have raised little Western support. Admiration for classical Greece, on the other hand, was almost universal and could raise sympathy for the case of the modern Greeks. Iblardi (talk) 13:32, 14 March 2009 (UTC)[reply]
I am not talking about a restored Byzantine Empire. I am talking about the nation of Greece becoming an empire rather than kingdom. The Empire of Greece or the Empire of the Hellenes. --Queen Elizabeth II's Little Spy (talk) 13:56, 14 March 2009 (UTC)[reply]
Yet I think that the reference to the Byzantine Empire, which was normally called the Greek Empire in the West even in the 19th century, would still have been too obvious. A mere kingdom lacks such a connotation. Iblardi (talk) 15:00, 14 March 2009 (UTC)[reply]
The king—emperor distinction is a West European notion. In other cultures there is usually a single monarchical title which is traditionally rendered into West European languages as an equivalent of either "king" or "emperor" based on a subjective judgement of a particular country's size and importance. Thus Chinese and Japanese monarchs are styled "emperors" in European languages while monarchs of smaller countries are mere "kings". In the case of Greece, the title used in 1832 was "basileus" – the same that was used by Byzantine rulers. It was the West Europeans who decided to translate the same title as "king" or "emeperor", depending on which entity – ancient or modern – they had in mind. There is a similar case with the Bulgarian and Russian "tsar". It comes from the name of Julius Ceasar, so should be considered an equivalent of "emperor"; but it is only in the case of Russia, not Bulgaria. — Kpalion(talk) 14:14, 14 March 2009 (UTC)[reply]
At that time "empire" had recently gained negative connotations among the Great Powers who were recogniziong the new state.--Wetman (talk) 18:16, 14 March 2009 (UTC)[reply]

Interesting discussion. I wonder why we don't refer to "Empress Victoria" and "Emperors" Edward VII, George V, Edward VIII and George VI. They were all Emperors of India, and head of the "British Empire", which included India, the UK, and lots of other places. -- JackofOz (talk) 21:28, 14 March 2009 (UTC)[reply]

Could it be Napoleon's fault? The word "emperor" implies a powerful monarch with an expansionist, often aggressive foreign policy, and it seems that, during the Napoleonic era, much of Europe (temporarily) developed a distaste for overly ambitious rulers. Britain, of course, had also long since ceased to be an absolute monarchy by that point, which could account for British monarchs' reluctance to use the word. "King" and "queen" are already loaded words; the name "Empress Victoria" would surely have raised some hackles in Parliament. --Fullobeans (talk) 22:38, 14 March 2009 (UTC)[reply]
And yet, Japan was always traditionally a very closed society, with virtually no foreign policy at all (other than "Foreigners are not welcome here"), until the late 19th century. It's also quite a small place, geographically. Yet it was never "King of Japan" but "Emperor of Japan". -- JackofOz (talk) 22:46, 14 March 2009 (UTC)[reply]
I'm speculating like a fiend here, but one could argue that, in a closed society with no foreign policy, the head of state is for all intents and purposes the ruler of the world. This could also account for the more liberal use of the word "empire" during the middle ages, when cowing your immediate neighbors made you a pretty big deal. I wonder also what the Japanese word for "emperor" is, and whether it's a direct translation or not. --Fullobeans (talk) 23:14, 14 March 2009 (UTC)[reply]
The word is tennō, and it is not a simple, straightforward translation. It is merely the best that could be done. Earlier it was mentioned that the Russian and Bulgarian emperors titles (Czar) came came from the name Julius Cæsar (=Kaiser), but the Roman title was Imperator, meaning something like autocrat. One is reversing things by believing that the head of an empire is an emperor; an empire is the entity that an emperor rules. As the tenno was (theoretically) the divine ultimate authority in Japan, "emperor" seemed the most appropriate translation, thus making Japan an "empire."
As for Napoleon giving the title a questionable odor, it didn't stop the dreikaiserbund.
B00P (talk) 02:23, 15 March 2009 (UTC)[reply]

Wasn't the title emperor by the middle ages in Western Europe came to be only associated with the Holy Roman Empire? And any imperial establishment would meet the opposition of Austria, the HRE's successor, and Russia. The German Empire had no Roman roots beside being influence by the HRE. And what did Iblardi mean by the unpopular Byzantine Empire? --Queen Elizabeth II's Little Spy (talk) 04:44, 15 March 2009 (UTC)[reply]

Yes, in the west "the Empire" was the HRE (but in the east it was Byzantium). The Holy Roman Emperor could claim responsibility for crowning kings even in far-off places like Cyprus. As for Iblardi's statement, the Byzantine Empire was considered a useless, decadent, corrupt state by Enlightenment and Romantic intellectuals. It was the bastard child of ancient Greece and Rome (which they usually considered ideal states). Adam Bishop (talk) 05:08, 15 March 2009 (UTC)[reply]
That is a bit harsh. But true, the Byzantine Empire was like it predecessor, the Roman Empire, corrupted in many ways with few righteous and good emperors. But what were the Greek's opinion on this? The last emperor Constantine XI was a considered a saint and national heroe to the Greeks. --Queen Elizabeth II's Little Spy (talk) 08:16, 15 March 2009 (UTC)[reply]

Ontario health insurance and mental health coverage

To what extent does the OHIP plan cover mental health services? Especially with respect to people 18 or 19 years old. 99.245.16.164 (talk) 08:21, 15 March 2009 (UTC)[reply]

Canadian equivalent of "Datuk"

My uncle who has lived over 70 years as a missionary in Malaysia, was recently awarded an award (either the Panglima Jasa Negara or the Panglima Setia Diraja) for his services in Malaysian education. He is therefore entitled to utilize Datuk before his name. I was not able to ascertain what exactly a Datuk equivalent would be in Canada when I referred to the Wiki article and if anyone does know I would appreciate it.142.68.216.139 (talk) 13:25, 15 March 2009 (UTC)[reply]